Anda di halaman 1dari 136
an7017 ‘Answer Key & Detaled Solutions — UPSC Civil Services Prelins Exam -2017 General Studies Paper 1 - INSIGHTS wy INSIGHTSIAS [osreensconmensa Home Feedback IAS Myths IAS Guide Daily Current Affairs Articles Daily Quiz Downloads Books Latest Posts ‘Answer Writing Challenges Toppers AIR Audio/Rajya Sabha TV Summaries Mains Self Study Tests - 2016 Mindmaps Offline Program MISSION IAS - 2017 Prelims 2017 Revision Tests MISSION 2018 = i-Learning 2017 PRELIMS ANSWER KEY / SUBSCRIBE TO UPDATES UPDATES / UPSC / UPSC 2017 Enter your email address to Receive Free Updates Join 134,556 other subscribers Answer Key & natn Detailed Sol utions — UPSC Insiohs PRELIMS TEST SERIES - 2018 Civil Services Prelims Exam tp iwmwinsightsonindia. com/2017/06/1S/answer-koy-upsc-civi-services-prolims-exam-2017-goreral-slusios-paper-t/ 196 an7017 ‘Answer Key & Detaled Solutions — UPSC Civil Services Prelins Exam -2017 General Studies Paper-1 - INSIGHTS -2017 General Studies Paper — 1 BY INSIGHTS * PUBLISHED JUNE 19, 2017 - UPDATED JUNE 19, 2017 Prelims Exam -2017 neral ies — 1 ET — Unknown 6 6 NOTE: Please share your scores in the comment box. If it’s your second attempt, share your previous score along with present score as well, 1) With reference to the Parliament of India, consider the following statements: 1. A private member's bill is a bill presented by a Member of Parliament who is not elected but only nominated by the President of India. 2. Recently, a private member's bill has been passed in the INSIGHTS MAINS TEST SERIES - 2017 TN os-yor gl oy INSIGHTS PRELIMS TEST SERIES ~ 2018 (OFFLINE — CLASSROOM) SUBSCRIBE Q To search type and hit enter RECENT POSTS. © [Insights Secure ~ 2017] UPSC Mains Questions on Current Affairs: 17 August 2017 August 17, 2017 © 9) “Education is the kindling of a flame, not the filling of a vessel.” Comment on Socrates’ view. August 17, 2017 © 8) “The measure of a man is what he does with power.” Do you agree with Plato's view? Comment. ‘August 17, 2017 tp iwmwinsightsonindia.com/2017/06/19/answer-koy-upsc-civi-services-pralims-exam-2017-general-studies-papor-1/ 21196 an7017 ‘Answer Key & Detaled Solutions — UPSC Civil Services Prelins Exam -2017 General Studies Paper 1 - INSIGHTS Parliament of India for the first time in its history. Which of the statements given above is/are correct? (a) t only (b) 2 only (c) Both 1 and 2 (d) Neither 1 nor 2 Justification: Statement 1: It is introduced by any member who is not a Minister. A bill introduced by a Minister is called as a public bill. Statement 2: Around 14 private member's bills have been passed since independence. http://www. prsindia.org/media/article citing-prs/only-14-private-members- bills-passed-since-independence- 3759/ 2) With reference to the difference between the culture of Rigvedic Aryans and Indus Valley people, © 7) The union cabinet recently approved the Code on Wages Bill, which proposes to make minimum wages a statutory right for all citizens. Discuss merits and demerits of this Bill August 17, 2017 © 6) Critically analyse the trends in energy scenario in the Middle East. Also examine what lessons could India learn from these countries in management of energy. ‘August 17, 2017 © 5) Templates for an upgraded rural health system have long been finalised and the Indian Public Health Standards were issued in 2007 and 2012, covering facilities from health sub-centres upwards. Despite these measures, why do you think tragedies such as recent Gorakhpur incident repeat? Critically comment. August 17, 2017 © 4) Its said that to improve ease of doing business in India, there is a need to provide viable alternatives to litigation and implement effective means for enforcement of contracts. Examine in detail why and what kind of institutional arbitration mechanism is required in India. ‘August 17, 2017 © 3) Do you think market-based health care is a fundamentally superior way to deliver health services to the poor? In the light of. NITI Aayog’s recent proposal to privatise government hospitals, critically examine. ‘August 17, 2017 tp iwmwinsightsonindia.com/2017/06/19/answer-koy-upsc-civi-services-pralims-exam-2017-general-studies-papor-1/ arta an7017 Answer Key & Detaled Solutions — UPSC Civil Services Prelims Exam -2017 General Studies Paper 1 - INSIGHTS which of the following statements is/are correct? 1. Rigvedic Aryans used the coat of mail and helmet in warfare whereas the people of Indus Valley Civilization did not leave any evidence of using them. 2. Rigvedic Aryans knew gold, silver and copper whereas Indus Valley people knew only copper and iron. 3. Rigvedic Aryans had domesticated the horse whereas there is no evidence of Indus Valley people having been aware of this animal. Select the correct answer using the code given below: (a) 1only (b) 2 and 3 only (©) 1 and 3 only (d) 1, 2 and 3 Solution: A Justification: Statement 2: The Indus valley people knew the use of copper, bronze, silver, gold but not iron. So, 2 is wrong. Statement 1: The Ramayana mentions chariots covered with © 2) In the light of recent events, critically ‘comment on the working of judiciary in South Asia. ‘August 17, 2017 © 1) In your opinion, which factors pushed North Korea to go nuclear? Critically examine. August 17, 2017 CATEGORIES Categories Select Category ’ ARCHIVES Archives Select Month ’ 5 6 7 8 9 40 41 12 13 14 15 16 17 «18 19 20 21 22 23 24 25 26 27 28 29 30 « May Jul» UPDATES As Mains approaches — Strategy and a few words. — Mittali Sethi 15 AUG, 2017 tp immwinsightsonindia.com/2017/06/19/answer-koy-upsc-civi-services-pralims-exam-2017-general-studies-papor-1/ arta an7017 ‘Answer Key & Detaled Solutions — UPSC Civil Services Prelins Exam -2017 General Studies Paper 1 - INSIGHTS leather. The Rig Vedic charioteers used varma (coats of mail) and sipra/sironastra (helmets). Equipped with asi (swords), hanas (arrows) and ilhianus (bows), the Kshatriyas on the chariots went to combat. Since statement 2 is wrong, given the options 1 must be correct. Statement 3: This is a disputed Statement. The Rig Veda mentions horse-drawn chariots with spoked wheels. But there is very little archaeological evidence of horses during the Harappan era and none at all earlier. The existence of the Harappan horse is a hotly disputed topic among archaeologists. Arguments: = Some claim that horse was NOT known. = Among the numerous seals found in Harappa there is none which represent a horse, while other animals like the bull, buffalo, and goat are represented. = Moreover, the cavalry (of horses) as an organised force can be Feedback from Insights Prelims Test Series 2017 Subscribers who have Cleared Prelims 2017 1 AUG, 2017 NEW INITIATIVE: Insights Learning (I-Learning) 31 JUL, 2017 RESULT: UPSC Civil Services IAS Prelims 2017 Result 27 UL, 2017 Update: How to Claim Your Refund — To Previous Year Paid Subscribers 19 JUL, 2017 UPDATES: Discounts on Online Test Series 2018 17 JUL, 2017 TOPPER'S STRATEGY: SHAURYA SUMAN, AIR 119 (CSE-2016), GS Marks 481, GS Strategy 15 JUL, 2017 UPDATES: Discounts on Online Test Series — 2018; Last Date for Offline Prelims Registration and Reading Rooms 11 JUL, 2017 MOTIVATION and IAS TOPPER’S Detailed STRATEGY, Notes: Dr Naveen Bhat, Rank 37, Second Attempt, Insights Student (Download His Full Notes) 10 JUL, 2017 tp iwmwinsightsonindia.com/2017/06/19/answer-koy-upsc-civi-services-pralims-exam-2017-general-studies-papor-1/ sit96 an7017 Answer Key & Detaled Solutions — UPSC Civil Services Prelims Exam -2017 General Studies Paper 1 - INSIGHTS traced only to the post vedic period, but not earlier. Arguments against: On the other hand, some scholars claim that the horse was widely domesticated and used in India in the area covered by the Indus- Sarasvati (or Harappan) Civilisation, but this evidence remains unsubstantiated. FINAL VIEW: But, based on the above, to say that “there is no evidence of Indus Valley people having been aware of this animal” is WRONG because there is no conclusive evidence that tells us whether they knew about horses, or whether they were aware of horses. We have insufficient evidence to claim anything about the connection between horses and Indus valley. Statement 3 is thus incorrect. http://www.thehindu.com/thehindu/o http://shodhganga .inflibnet.ac.in/bitst https://www.telegraphindia.com/115¢ IAS TOPPER'S STRATEGY: Santhosh BM, Rank 307, Third Attempt, Insights Offline Student 9 JUL, 2017 MOTIVATIONAL POSTS MOTIVATION: An open letter to those who couldn't clear Prelims — Mittali Sethi, Rank 56, CSE-2016 28 JUL, 2017 MOTIVATION: Emotional Intelligence 17 JUL, 2017 Motivation & Topper’s Strategy: UPSC IAS Topper NANDINI K R, Rank - 1, CSE 2016, Insights Offline Student 4 JUL, 2017 MOTIVATION: Nikhil B, Rank 107, Second Attempt, Essay Score 45 to 158, Insights Offline Student 4UL, 2017 MOTIVATION: Emotional Intelligence 28 JUN, 2017 MOTIVATION: Three Failed Prelims, Rank 287 — Tips for Prelims 17 JUN, 2017 RECENT CURRENT EVENTS POSTS Insights Daily Current Affairs, 16 Aug 2017 16 AUG, 2017 tp iwmwinsightsonindia.com/2017/06/19/answer-koy-upsc-civi-services-pralims-exam-2017-general-studies-papor-1/ arta an7017 Answer Key & Detaled Solutions — UPSC Civil Services Prelims Exam -2017 General Studies Paper 1 - INSIGHTS 3. Recognition of Prior Learning Scheme’ is sometimes mentioned in the news with reference to (a) Certifying the skills acquired by construction workers through traditional channels. (b) Enrolling the persons in Universities for distance learning programmes. (c) Reserving some skilled jobs to rural and urban poor in some public sector undertakings. (d) Certifying the skills acquired by trainees under the National Skill Development Programme. Solution: A Why: Aspirants cover core schemes and programmes but often miss covering ancillary and peripheral aspects. This was picked from ‘between the lines’ on the official website of PMKVY. Such ancillary questions can be expected even in the coming years. Justification: Option A: The ‘Recognition of Prior Learning’ scheme — underway across construction sites in five states — to Insights Daily Current Affairs, 15 Aug 2017 415 AUG, 2017 Insights Daily Current Affairs, 14 Aug 2017 14 AUG, 2017 Insights Daily Current Affairs, 12 Aug 2017 12 AUG, 2017 Insights Daily Current Affairs, 11 Aug 2017 11 AUG, 2017 RECENT SECURE — 2016 POSTS [Insights Secure — 2017] UPSC Mains Questions on Current Affairs: 17 August 2017 17 AUG, 2017 SECURE SYNOPSIS: 14 AUGUST 2017 17 AUG, 2017 [Insights Secure — 2017] UPSC Mains Questions on Current Affairs: 16 August 2017 16 AUG, 2017 [Insights Secure - 2017] UPSC Mains Questions on Current Affairs: 15 August 2017 415 AUG, 2017 CURRENT EVENTS QUIZ tp immwinsightsonindia.com/2017/06/19/answer-koy-upsc-civi-services-pralims-exam-2017-general-studies-papor-1/ 7196 an7017 Answer Key & Detaled Solutions — UPSC Civil Services Prelims Exam -2017 General Studies Paper 1 - INSIGHTS certify skills acquired by workers through traditional learning channels. = The project may be of particular relevance to a country where just 2 per cent of the workforce is certified as skilled. Most deemed to be outside the skilled category in India are those who have typically picked up a skill while on the job, without any formal degree to back this up. Option D is incorrect since it talks about trainees who are not fully skilled to get recognition of any prior learning. Individuals with prior learning experience or skills will also be assessed and certified under Recognition of Prior Learning (RPL) in Pradhan Mantri Kaushal Vikas Yojana (PMKVY), but this isn’t applicable to trainees. So, D is incorrect. http: //www.skilldevelopment.gov.in/p 4, From the ecological point of view, which one of the following assumes importance in being a QUIZ — 2017: Insights Current Affairs Quiz, 16 August 2017 QUIZ — 2017: Insights Current Affairs Quiz, 15 August 2017 QUIZ — 2017: Insights Current Affairs Quiz, 14 August 2017 QUIZ - 2017: Insights Current Affairs Quiz, 11 August 2017 MINDMAPS Insights MINDMAPS:"India- Philippines Relations” and “Student Activism and Politics”. Insights MINDMAPS: “DNA Profiling in India” and “Medical Termination of Pregnancy Act”. Insights MINDMAPS: “75 years of the Quit India Movement” and “Privatising Public Hospitals”. Insights MINDMAPS: “India — US Cooperation in S&T” and “Outer Space Treaty (OST)”. RAJYASABHA TV NEWS DEBATES. Policy Watch Episode - 279 : Govt sops for stressed telcos 16 AUG, 2017 tp iwmwinsightsonindia.com/2017/06/19/answer-koy-upsc-civi-services-pralims-exam-2017-general-studies-papor-1/ arta an7017 ‘Answer Key & Detaled Solutions — UPSC Civil Services Prelins Exam -2017 General Studies Paper 1 - INSIGHTS good link between the Eastern Ghats and the Western Ghats? (a) Sathyamangalam Tiger Reserve (b) Nallamala Forest (©) Nagarhole National Park (d) Seshachalam Biosphere Reserve Solution: A Why: UPSC has the habit of picking up options from questions of its past year papers and asking new questions after improvising them. Sathyamangalam Tiger Reserve can be found occasionally mentioned in CSP and CAPF papers; hence the question. Justification: Sathyamangalam forest range is a significant wildlife corridor in the Nilgiri Biosphere Reserve between the Western Ghats and the rest of the Eastern Ghats and a genetic link between the four other protected areas which it adjoins, including the Billigiriranga Swamy Temple Wildlife Sanctuary, Sigur Plateau, Mudumalai National Park and Bandipur National Park. Map below shows the protected area. https://en.wikipedia.org/wiki/Sathyarr Policy Watch Episode - 279 : Govt sops for stressed telcos 16 AUG, 2017 INSIGHTS INTO EDITORIAL Insights into Editorial : Doklam, China’s Strategic Calculus and India’s Policy Options Insights into Editorial : Let’s talk about a supplemental income Insights into Editorial : Nitrogen Overload Insights into Editorial : Behind India’s unease with a global child abduction law TOPPER'S STRATEGIES As Mains approaches — Strategy and a few words. — Mittali Sethi POLITICAL SCIENCE AND INTERNATIONAL RELATIONS OPTIONAL ‘STRATEGY’, Aishwarya Dongre, AIR 196 LAW OPTIONAL STRATEGY : Pratik Tayal , AIR 92 CSE 2016 TOPPER’S STRATEGY: SHAURYA. SUMAN, AIR 119 (CSE-2016), GS Marks 481, GS Strategy tp iwmwinsightsonindia.com/2017/06/19/answer-koy-upsc-civi-services-pralims-exam-2017-general-studies-papor-1/ oita8 an7017 ‘Answer Key & Detaled Solutions — UPSC Civil Services Prelins Exam -2017 General Studies Paper 1 - INSIGHTS oo ge 5. One of the implications of equality in society is the absence of (a) Privileges (b) Restraints (c) Competition (d) Ideology Solution: A Justification: Article 18 of the constitution justifies this. Under Right to equality, A18 abolishes titular privileges (except military or academic) granted to citizens of India. MOTIVATION and IAS TOPPER’S Detailed STRATEGY, Notes: Dr Naveen Bhat, Rank 37, Second Attempt, Insights Student (Download His Full Notes) TAS TOPPER’S STRATEGY: Santhosh B M, Rank 307, Third Attempt, Insights Offline Student PSYCHOLOGY STRATEGY for Civil Services: Strategy, Booklist, Resources and Notes- Mittali Sethi AIR 56 (302 Marks) TOPPER’S STRATEGY: Meghashree D R, Rank 289, CSE - 2016, No Coaching, Insights Offline Student TOPPER’S STRATEGY: Shabarish P, Rank 617, CSE-2016, Insights Offline Student. TOPPER’S STRATEGY: Ann Mary George, Rank 123 — Didn’t clear prelims in first two attempts, scored 137 in third prelims RECENT ESSAY CHALLENGES Insights Weekly Essay Challenges 2017 — Week 31: Interrelationship between Gender Equality and Sustainable Development Insights Weekly Essay Challenges 2017 - Week 30: Utility and tp iwmwinsightsonindia. com/2017/06/1S/answer-koy-upsc-civ-services-prolims-exam-2017-goreral-slusios-paper-t/ rorta6 an7017 ‘Answer Key & Detaled Solutions — UPSC Civil Services Prelins Exam -2017 General Studies Paper 1 - INSIGHTS 6) Consider the following statements in respect of Trade Related Analysis of Fauna and Flora in Commerce (TRAFFIC): 1. TRAFFIC is a bureau under United Nations Environment Programme (UNEP). 2. The mission of TRAFFIC is to ensure that trade in wild plants and animals is not a threat to the conservation of nature. Which of the above statements is/are correct? (a) 1 only (b) 2 only (© Both 1 and 2 (d) Neither 1 nor 2 Solution: B Why: WWF and related agencies/schemes/initiatives are a hot favourite of UPSC, one example being Earth hour. Try to cover WWF website comprehensively to answer such questions. Justification: Statement 1: It is a NGO, not a bureau under UNEP. So, 1 is wrong. relevance of Parliament in our polity Insights Weekly Essay Challenges 2017 — Week 29: Caste System — Source of India’s Eternal Inequality? Insights Weekly Essay Challenges 2017 - Week 28: Indian Democracy, Media and Public Opinion — Does Public Opinion Matter in Policymaking? Insights Weekly Essay Challenges 2017 — Week 27: Poverty and Environment — Their Interrelationship is the Key to Sustainable World MUST READ BOOKS 1) Introduction to the Constitution of India 20 Edition 2) India Since Independence - Bipan Chandra 3) India's Struggle For Independence — Bipan Chandra 4) Indian Economy : For Civil Services Examinations 6th Edition (2014) - Ramesh Singh 5) Indian Polity for Civil Services Examinations 4th Edition - Laxmikanth 6)Geography of India- Majid Hussain - Sth Edition 7) Ethics In Governance tp iwmwinsightsonindia.com/2017/06/1S/answer-koy-upsc-civi-services-pralims-exam-2017-general-stucies-papor-1/ a6 an7017 Answer Key & Detaled Solutions — UPSC Civil Services Prelims Exam -2017 General Studies Paper 1 - INSIGHTS TRAFFIC is governed by the TRAFFIC Committee, a steering group composed of members of TRAFFIC’s partner organizations, WWF and IUCN. Statement 2: It is working globally on Wildlife trade monitoring network, It specializes in investigating and analysing wildlife trade trends, patterns, impacts and drivers to provide the leading knowledge base on trade in wild animals and plants. http: //www.traffic.org/overview 7. Which principle among the following was added to the Directive Principles of Stat Policy by the 42nd Amendment to the Constitution? (a) Equal pay for equal work for both men and women (b) Participation of workers in the management of industries (©) Right to work, education and public assistance 8)India After Gandhi: The History Of The World's Largest Democracy 9) Mastering Modern World History -Sth Edition, Norman Lowe 10) Select Constitutions: 16th Edition - S Chand 11) Challenge And Strategy : Rethinking India’s Foreign Policy - Rajiv Sikri 12) India's Foreign Policy - Muchkund Dubey 13) History of the World from the Late Nineteenth to the Early Twenty First Century ~ Arjun Dev 14) Social Problems in India 2 Edition - Ram Ahuja 15) Politics and Ethics of the Indian Constitution 16) Public Institutions in India: Performance and Design - PRATAP BHANU MEHTA, 17) Certificate Physical and Human Geography - Goh Cheng Leong 18) India’s National Security — A Reader 19) Indian Social System - Ram Ahuja tp iwmwinsightsonindia.com/2017/06/1S/answer-koy-upsc-civi-services-pralims-exam-2017-general-stucies-papor-1/ tarts an7017 ‘Answer Key & Detaled Solutions — UPSC Civil Services Prelins Exam -2017 General Studies Paper 1 - INSIGHTS (d) Securing living wage and human conditions of work to workers Solution: B Justification: The 42nd Amendment added new Directive Principles, viz Article 39A, Article 43A and Article 48A. Article 43A deals with ‘Participation of workers in management of industries’. http://indiacode.nic.in/coiweb/amend, 8. Which one of the following statements is correct? (a) Rights are claims of the State against the citizens. (b) Rights are privileges which are incorporated in the Constitution of a State, (c) Rights are claims of the citizens against the State. (d) Rights are privileges of a few citizens against the many. Solution: C tp iwmwinsightsonindia.com/2017/06/1S/answer-koy-upsc-civi-services-pralims-exam-2017-general-stucies-papor-1/ rarta6 anra017 ‘Answer Key & Detaled Solutions — UPSC Civil Services Prelins Exam -2017 General Studies Paper-1 - INSIGHTS Justification: Option C: The philosophy of rights is grounded in preventing the exploitation of the citizen by the state. So, essentially it is the claim of an individual against the state, Option B: This is wrong because rights are not privileges, they are the bare minimum for the formation of a democratic state. Also, the constitution abolishes the concept of special privileges. 9. Which of the following gives ‘Global Gender Gap Index’ ranking to the countries of the world? (a) World Economic Forum (b) UN Human Rights Council (©) UN Women (d) World Health Organization Solution: A Why: India was ranked at a lowly 87th position in the gender gap index that created headlines. It is an tp iwmwinsightsonindia.com/2017/06/1S/answer-koy-upsc-civi-services-pralims-exam-2017-general-stucies-papor-1/ saitas an7017 Answer Key & Detaled Solutions — UPSC Civil Services Prelims Exam -2017 General Studies Paper 1 - INSIGHTS important report brought out by WEF. Justification: http://reports.weforum.org/global- gender-gap-report-2016/rankings/ 10) Which of the following statements is/are correct regarding Smart India Hackathon 2017? 1. It is a centrally sponsored scheme for developing every city of our country into Smart Cities in a decade, . It is an initiative to identify new digital technology innovations for solving the many problems faced by our country. . It is a programme aimed at making all the financial transactions in our country completely digital in a decade. N w Select the correct answer using the code given below : (a) 1 and 3 only (b) 2 only (c) 3 only tp iwmwinsightsonindia.com/2017/06/1S/answer-koy-upsc-civi-services-pralims-exam-2017-general-stucies-papor-1/ tsita6 an7017 Answer Key & Detaled Solutions — UPSC Civil Services Prelims Exam -2017 General Studies Paper 1 - INSIGHTS (d) 2 and 3 only Solution: B Why: It is a part of Digital India programme of the government. Expect related questions from official events and schemes. Justification: Statement 1 and 3: There are no such provisions. Statement 2: It was a digital product development competition during which teams of thousands of technology students built innovative digital solutions for the problems posted by different central govt. ministries/ departments. http://pib.nic.in/newsite/PrintRelease. relid=159825 11) Which of the following statements is/are correct regarding the Monetary Policy Committee (MPC)? 1, It decides the RBI's benchmark interest rates. 2. It is a 12-member body including the Governor of RBI and is reconstituted every year. tp iwmwinsightsonindia.com/2017/06/1S/answer-koy-upsc-civi-services-pralims-exam-2017-general-stucies-papor-1/ teita6 anra017 Answer Key & Detaled Solutions — UPSC Civil Services Prelims Exam -2017 General Studies Paper 1 - INSIGHTS 3. It functions under the chairmanship of the Union Finance Minister. Select the correct answer using the code given below : (a) Lonly (b) 1 and 2 only (c) 3 only (d) 2 and 3 only Solution: A Why: It was a major news break, and perhaps the most significant reform in India’s monetary policy framework in past few decades. Justification: Statement 1: It decides the repo rates, CRR, SLR etc, It replaced the earlier system of policy rate veto by the RBI governor. Statement 2: It consists of six members (RBI Governor, Deputy Gov RBI, One RBI officer and three members nominated by the government), and they hold office for a period of four years. Statement 3: The Governor of the RBI is the ex-officio Chairperson of MPC, tp iwmwinsightsonindia.com/2017/06/1S/answer-koy-upsc-civi-services-pralims-exam-2017-general-stucies-papor-1/ wits an7017 ‘Answer Key & Detaled Solutions — UPSC Civil Services Prelins Exam -2017 General Studies Paper 1 - INSIGHTS http://pib.nic.in/newsite/PrintRelease. relid=151264 12) With reference to Manipuri Sankirtana, consider the following statements: . It is a song and dance performance. . Cymbals are the only musical instruments used in the performance. . It is performed to narrate the life and deeds of Lord Krishna. N w Which of the statements given above is/are correct? (a) 1, 2 and 3. (b) 1 and 3 only (c) 2 and 3 only (d) 1 only Solution: B Why: Sankirtana is one of the few important Indian heritages inscribed on the UNESCO Representative List of the Intangible Cultural Heritage of Humanity. tp iwmwinsightsonindia.com/2017/06/1S/answer-koy-upsc-civi-services-pralims-exam-2017-general-stucies-papor-1/ tarta6 an7017 ‘Answer Key & Detaled Solutions — UPSC Civil Services Prelins Exam -2017 General Studies Paper 1 - INSIGHTS Justification: Statement 1 and 3: Sankirtana performers narrate the lives and deeds of Krishna through ritual singing, drumming and dancing of Manipur. So, both 1 and 3 are correct. Statement 2: Since drums are also used, statement 2 is incorrect. https://ich.unesco.org/en/RL/sankirta ritual-singing-drumming-and- dancing-of-manipur-00843 13) Who among the following was/were associated with the introduction of Ryotwari Settlement in India during the British rule? 1. Lord Cornwallis 2. Alexander Read 3. Thomas Munro Select the correct answer using the code given below: (a) 1 only (b) 1 and 3 only (©) 2 and 3 only (d) 1, 2and3 tp iwmwinsightsonindia.com/2017/06/1S/answer-koy-upsc-civi-services-pralims-exam-2017-general- studies papor-1/ s9/ta6 an7017 Answer Key & Detaled Solutions — UPSC Civil Services Prelims Exam -2017 General Studies Paper 1 - INSIGHTS Solution: C Justification: Statement 1 and 3: The Ryotwari system was devised by Capt. Alexander Read and Thomas Munro at the end of the 18th century. It was later introduced by Thomas Munro when he was governor (1820-27) of Madras. Statement 2: He is associated with the Cornwallis Code 1793 and could not have been the answer since his tenure preceded the ryotwari settlement regime. The principle behind Roytwari was the direct collection of the land revenue from each individual cultivator by government agents. https://www.britannica.com/topic/ryo system 14) In the context of solving pollution problems, what is/are the advantage/advantages of bioremediation technique? 1, It is a technique for cleaning up pollution by enhancing the same tp iwmwinsightsonindia.com/2017/06/1S/answer-koy-upsc-civi-services-pralims-exam-2017-general-stucies-papor-1/ 20/196 anra017 tp iwmwinsightsonindia.com/2017/06/1S/answer-koy-upsc-civi-services-pralims-exam-2017-general-stucies-papor-1/ Answer Key & Detaled Solutions — UPSC Civil Services Prelims Exam -2017 General Studies Paper 1 - INSIGHTS biodegradation process that occurs in nature. . Any contaminant with heavy metals such as cadmium and lead can be readily and completely treated by bioremediation using microorganisms. . Genetic engineering can be used to create microorganisms specifically designed for bioremediation. N w Select the correct answer using the code given below: (a) 1 only (b) 2 and 3 only (©) 1 and 3 only (d) 1, 2. and 3 Solution: C Why: Expect around 6-8 standard conceptual questions on Eco- engineering every year. Bioremediation, biofuels, genetic engineering are some such standard concepts. Justification: Statement 1: Bioremediation uses naturally occurring organisms to break down hazardous substances into less toxic or non toxic substances. aves an7017 ‘Answer Key & Detaled Solutions - UPSC Gil Series Palins Exam -2017 General Stsies Paper-1 - INSIGHTS Statement 2: Always be cautious of extreme statements and words as we have repeatedly pointed out in our 2017 Prelims Test Series. Completely and readily (quickly) are such words here, Bio-remediation is a slow process, so, Heavy metals such as cadmium and lead are NOT readily absorbed or captured by microorganisms. UPSC had lifted this straight from Wikipedia. Statement 3: This will be a standard practice in the industry in coming years. For e.g. Alcanivorax, a bacteria with oil-eating abilities, can be used to create species that are much more capable of cleaning oil spills. https://www.sciencedaily.com/release https://www.scientificamerican.com/a microbes-clean-up-oil-spills/ 15. The Trade Disputes Act of 1929 provided for (a) the participation of workers in the management of industries. tp iwmwinsightsonindia.com/2017/06/1S/answer-koy-upsc-civi-services-pralims-exam-2017-general-stucies-papor-1/ ree anra017 ‘Answer Key & Detaled Solutions — UPSC Civil Services Prelins Exam -2017 General Studies Paper 1 - INSIGHTS (b) arbitrary powers to the management to quell industrial disputes, (c) an intervention by the British Court in the event of a trade dispute. (d) a system of tribunals and a ban on strikes, Solution: D Why: Labour reforms were hitherto largely ignored by UPSC, and only occasionally covered in NDA, CDS. and CAPF papers in the form of Whitley Commission (employee- employer dispute resolution) and labour uprisings. Now, UPSC is moving to peripheral areas in history. Justification: Prior to the year 1947 (India’s Independence), The Trade Disputes Act, 1929 used to settle industrial disputes. Tribunals: The main object of the 1929 Act was to make provisions for establishment of Courts of Inquiry and Boards of Conciliation with a view to investigate and settle trade disputes. Strikes: The Act prohibited strikes or lock-outs without notice in public tility services; it also made any strike or lock-out illegal which had tp iwmwinsightsonindia.com/2017/06/1S/answer-koy-upsc-civi-services-pralims-exam-2017-general- studies papor-1/ arias an7017 Answer Key & Detaled Solutions — UPSC Civil Services Prelims Exam -2017 General Studies Paper 1 - INSIGHTS any object other than the furtherance of a trade dispute within the trade or industry. http://shodhganga.inflibnet.ac.in/bitst 16. Local self-government can be best explained as an exercise in (a) Federalism (b) Democratic decentralisation (c) Administrative delegation (d) Direct democracy Solution: B Option B: Decentralization and grass roots empowerment are the core guiding principles of local self- government as under 73rd and 74th constitutional amendment. For e.g. Gram Panchayats can sanction sanitation and welfare projects on their own. Option A: Local bodies are not federal entities as is the position of states in India. tp iwmwinsightsonindia.com/2017/06/1S/answer-koy-upsc-civi-services-pralims-exam-2017-general-stucies-papor-1/ 2arta6 anra017 Answer Key & Detaled Solutions — UPSC Civil Services Prelims Exam -2017 General Studies Paper 1 - INSIGHTS Option C: This isn’t administrative delegation because local self- government bodies are political entities that are elected directly (or indirectly) by people and managed by their representatives, Option D: Since it involves election and indirect decision making by people's representatives, there can be no direct democracy (where all decisions are made directly by the people). 17. Consider the following statements: With reference to the Constitution of India, the Directive Principles of State Policy constitute limitations upon 1. legislative function. 2. executive function. Which of the above statements is/are correct? (a) 1 only (b) 2 only (c) Both 1 and 2 tp iwmwinsightsonindia.com/2017/06/1S/answer-koy-upsc-civi-services-pralims-exam-2017-general-stucies-papor-1/ 2596 anra017 ‘Answer Key & Detaled Solutions — UPSC Civil Services Prelins Exam -2017 General Studies Paper 1 - INSIGHTS (d) Neither 1 nor 2 Solution: D Justification: It is the fundamental rights that constitute limitations upon state action (whether legislative or executive). The directive principles are in the nature of instruments of instruction to the government of the day to achieve certain ends by their actions. It guides them, not restricts them. 18. The term ‘Digital Single Market Strategy’ seen in the news refers to (a) ASEAN (b) BRICS (©) EU (a) G20 Solution: C Why: Brexit from Euro Zone brought focus on EU amid impending Global Digital revolution. UPSC lifted this from EU's official website. tp iwmwinsightsonindia.com/2017/06/1S/answer-koy-upsc-civi-services-pralims-exam-2017-general-stucies-papor-1/ 20/196 ani7a0%7 ‘Answer Key & Detailed Solutions - UPSC Chil Services Prelims Exam -2017 General Studies Paper- 1 - INSIGHTS Justification: The Digital Single Market is part of the Digital Agenda for Europe 2020 program of the EU, an initiative of Europe 2020 proposed strategy. The three said “pillars” of the European Commission strategy are: = Access to online products and services = Conditions for digital networks and services to grow and thrive = Growth of the European digital economy https://ec.europa.eu/commission/prio single-market_en 19. At one of the places in India, if you stand on the seashore and watch the sea, you will find that the sea water recedes from the shore line a few kilometres and comes back to the shore, twice a day, and you can actually walk on the sea floor when the water recedes. This unique phenomenon is seen at (a) Bhavnagar tp iwmwinsightsonindia.com/2017/06/1S/answer-koy-upsc-civi-services-pralims-exam-2017-general-stucies-papor-1/ 2796 an7017 Answer Key & Detaled Solutions — UPSC Civil Services Prelims Exam -2017 General Studies Paper 1 - INSIGHTS (b) Bheemunipatnam (c) Chandipur (d) Nagapattinam Solution: C Why: Chandipur-on-sea and Gopalpur-on-sea are frequently mentioned in past year papers of UPSC. This question is an improvisation of these places mentioned in the options of those papers. Justification: Chandipur-on-sea (Odisha) beach is unique in that the water recedes up to 5 kilometers during the ebb tide. You can ‘literally’ walk into the beach as the sea water recedes away from the shore line during the ebb tide (time period between the high tide and the low tide). https://en.wikipedia.org/wiki/Chandip 20) With reference to the ‘Prohibition of Benami Property Transactions Act, tp iwmwinsightsonindia.com/2017/06/1S/answer-koy-upsc-civi-services-pralims-exam-2017-general-stucies-papor-1/ 2aria6 srraot7 Answer Key & DetoledSolstons -UPSC Chil Sences Prelims Exam 2017 General Stes Paper-1 = INSIGHTS 1988 (PBPT Act)’, consider the following statements 1, A property transaction is not treated as a benami transaction if the owner of the property is not aware of the transaction. . Properties held benami are liable for confiscation by the Government. 3. The Act provides for three authorities for investigations but does not provide for any appellate mechanism. N Which of the statements given above is/are correct? (a) 1only (b) 2 only (c) 1 and 3 only (d) 2 and 3 only Solution: B Why: Current Events: Benami Property Transactions Act, 1988 has been amended by the Benami Transactions (Prohibition) Amendment Act, 2016 (BTP Amendment Act). Justification: Statement 1: “Ignorance of law cannot be an excuse for the violation of law” is a tp iwmwinsightsonindia.com/2017/06/1S/answer-koy-upsc-civi-services-pralims-exam-2017-general- studies papor-1/ 20/196 an7017 ‘Answer Key & Detaled Solutions — UPSC Civil Services Prelins Exam -2017 General Studies Paper-1 - INSIGHTS basic principle based on which statement 1 is wrong. Statement 2: Properties held benami are liable for confiscation by the Government without payment of compensation. Statement 3: An appellate mechanism has been provided under the PBPT Act in the form of Adjudicating Authority and Appellate Tribunal. http://pib.nic.in/newsite/PrintRelease, relid=153085 21) Due to some reasons, if there is a huge fall in the population of species of butterflies, what could be its likely consequence/consequences? . Pollination of some plants could be adversely affected. . There could be a drastic increase in the fungal infections of some cultivated plants. . It could lead to a fall in the population of some species of wasps, spiders and birds. N w tp iwmwinsightsonindia.com/2017/06/1S/answer-koy-upsc-civi-services-pralims-exam-2017-general- studies papor-1/ sorta6 an7017 ‘Answer Key & Detaled Solutions — UPSC Civil Services Prelins Exam -2017 General Studies Paper-1 - INSIGHTS Select the correct answer using the code given below: (a) Lonly (b) 2 and 3 only (c) 1 and 3 only (d) 1, 2. and 3 Solution: C Statement 1: Bees and butterflies play a vital role in the pollination of plants and the production of crops by transporting pollen grains from one place to another. Statement 2: This is an absurd statement. Statement 3: These are some of the common predators of butterflies: wasps, ants, parasitic flies, birds, snakes, toads, rats etc. The decline in butterfly population would therefore adversely affect the food chain. http://www. thebutterflysite.com/wha eats- butterflies.shtml#sthash. PPoslJj4.dput tp iwmwinsightsonindia.com/2017/06/1S/answer-koy-upsc-civi-services-pralims-exam-2017-general-stucies-papor-1/ sivta6 anra017 ‘Answer Key & Detaled Solutions - UPSC Gil Series Palins Exam -2017 General Stsies Paper-1 - INSIGHTS 22) It is possible to produce algae based biofuels, but what is/are the likely limitation(s) of developing countries in promoting this industry? 1. Production of algae based biofuels is possible in seas only and not on continents. . Setting up and engineering the algae based biofuel production requires high level of expertise/technology until the construction is completed. . Economically viable production necessitates the setting up of large scale facilities which may raise ecological and social concerns, N w Select the correct answer using the code given below: (a) 1 and 2 only (b) 2 and 3 only (c) 3 only (d) 1, 2 and 3 Solution: B Justification: Statement 1: Algaculture (farming algae) can be initiated on land unsuitable for agriculture or saline water or wastewater. So, 1 is wrong. tp iwmwinsightsonindia.com/2017/06/1S/answer-koy-upsc-civi-services-pralims-exam-2017-general-stucies-papor-1/ sartae an7017 ‘Answer Key & Detaled Solutions — UPSC Civil Services Prelins Exam -2017 General Studies Paper 1 - INSIGHTS Statement 2: Producing algae for bio- fuels (excluding simple seaweed production) requires significant capital investments, which may be a significant barrier in developing countries that have a weak investment climate, = Producing biofuels from algae is still in a pre-commercial state of technology development. Most production plants are therefore prototypes, and the development and engineering of such plants requires a high level of expertise. Statement 3: Using the land allotted to food crops to algal biofuel reduces the amount of food available for humans, resulting in an increased cost for both the food and the fuel produced. This is an ecological as well as economic concern. http://www.climatetechwiki.org/techn 23) Which of the following are the objectives of ‘National Nutrition Mission’? 1. To create awareness relating to malnutrition among pregnant tp immwinsightsonindia.com/2017/06/1S/answer-koy-upsc-civi-services-pralims-exam-2017-general-studiespapor-1/ sartas an7017 tpn ‘Answer Key & Detaled Solutions - UPSC Gil Series Palins Exam -2017 General Stsies Paper-1 - INSIGHTS women and lactating mothers. 2. To reduce the incidence of anaemia among young children, adolescent girls and women. 3. To promote the consumption of millets, coarse cereals and unpolished rice. 4, To promote the consumption of poultry eggs. Select the correct answer using the code given below: (a) 1 and 2 only (b) 1, 2. and 3 only (©) 1, 2 and 4 only (d) 3 and 4 only Solution: A Why: Picked from the recent schemes section of the Ministry of Women and Child Development Website. Justification: Statement 3 and 4 are wrong because there is no mention of eggs or unpolished rice or millets in the official release document: http://wed.nic.in/sites/default/files/Mt http://wed.nic.in/documents/national- nutrition-mission-nnm insightsonindia. com2017/06/19/answer-koy-upsc-civi-services-pralims-exam-2017-general-studies-paper-1/ sartas ani7a0%7 ‘Answer Key & Detailed Solutions - UPSC Chil Services Prelims Exam -2017 General Studies Paper- 1 - INSIGHTS Statement 1 and 2: There are two components of the National Nutrition Mission as follows: 1. Information, Education and Communication (IEC) Campaign against malnutrition 2. Multi-sectoral Nutrition Programme The key objectives of the programmes under the Mission are as under: = To create awareness relating to malnutrition amongst pregnant women, lactating mothers, promote healthy lactating practices and importance of balanced nutrition; To improve maternal and child under-nutrition in 200 high burdened districts and to prevent and reduce the under-nutrition prevalent among children below 3 years; To reduce incidence of anaemia among young children, adolescent girls and women. http://pib.nic.in/newsite/PrintRelease. relid=108509 tp iwmwinsightsonindia.com/2017/06/1S/answer-koy-upsc-civi-services-pralims-exam-2017-general- studies papor-1/ ssita6 an7017 Answer Key & Detaled Solutions — UPSC Civil Services Prelims Exam -2017 General Studies Paper 1 - INSIGHTS 24) Consider the following statements: 1. The Factories Act, 1881 was passed with a view to fix the wages of industrial workers and to allow the workers to form trade unions. . N.M. Lokhande was a pioneer in organizing the labour movement in British India. N Which of the above statements is/are correct? (a) Lonly (b) 2 only (c) Both 1 and 2 (d) Neither 1 nor 2 Solution: B Statement 1: It is wrong. The 1881 Act aimed to improve working condtions of labour. It prohibited the employment of children under the age of seven, limited the number of working hours for children below the age of twelve and required that dangerous machinery should be fenced properly. tp iwmwinsightsonindia.com/2017/06/1S/answer-koy-upsc-civi-services-pralims-exam-2017-general-stucies-papor-1/ a6ita6 an7017 ‘Answer Key & Detaled Solutions - UPSC Givi Series Palins Exam -2017 General Stsies Paper~1 - INSIGHTS Statement 2: Lokhande was a pioneer of the labour movement in India. He is remembered not only for ameliorating the working conditions of textile mill-hands in the 19th century but also for his courageous initiatives on caste and communal issues. http://tinyurl.com/y7jy5ybj https://en.wikipedia.org/wiki/Narayan 25) In the context of mitigating the impending global warming due to anthropogenic emissions of carbon dioxide, which of the following can be the potential sites for carbon sequestration? 1, Abandoned and uneconomic coal seams 2. Depleted oil and gas reservoirs 3. Subterranean deep saline formations Select the correct answer using the code given below: (a) 1 and 2 only (b) 3 only (c) 1 and 3 only tp iwmwinsightsonindia.com/2017/06/1S/answer-koy-upsc-civi-services-pralims-exam-2017-general- studies papor-1/ arita6 an7017 ‘Answer Key & Detaled Solutions — UPSC Civil Services Prelins Exam -2017 General Studies Paper 1 - INSIGHTS (d) 1, 2and3 Solution: D Geo-sequestration is a method that involves injecting carbon dioxide, generally in supercritical form, directly into underground geological formations, Statement 1: Unmineable coal seams can be used to store CO2 because the CO2 molecules attach to the surface of coal. Statement 2: CO2 is sometimes injected into declining oil fields to increase oil recovery. Other potential sites are gas fields, saline formations (statement 3), unmineable coal seams, and saline- filled basalt formations can act as storage sites. After sequestration, various physical (e.g., highly impermeable caprock) and geochemical trapping mechanisms prevent the CO2 from escaping to the surface. tp iwmwinsightsonindia.com/2017/06/1S/answer-koy-upsc-civi-services-pralims-exam-2017-general-stucies-papor-1/ serta6 an7017 Answer Key & Detaled Solutions — UPSC Civil Services Prelims Exam -2017 General Studies Paper 1 - INSIGHTS 26. The object of the Butler Committee of 1927 was to? (a) Define the jurisdiction of the Central and Provincial Governments. (b) Define the powers of the Secretary of State for India. (c) Impose censorship on national press, (d) Improve the relationship between the Government of India and the Indian States. Solution: D Justification: It was also known as the Indian states committee. Relations between the Indian Princes and the crown were not well defined because the extent of sovereignty of the Paramount power was not settled properly, The Indian states committee (under the Chairmanship of Harcourt Butler) in 1927 was appointed to investigate and clarify the relationship between the paramount power and the Princes. http://tinyurl.com/yatpetz2 tp iwmwinsightsonindia.com/2017/06/1S/answer-koy-upsc-civi-services-pralims-exam-2017-general- studies papor-1/ s9rta6 anra017 Answer Key & Detaled Solutions — UPSC Civil Services Prelims Exam -2017 General Studies Paper 1 - INSIGHTS 27. The term ‘Domestic Content Requirement’ is sometimes seen in the news with reference to (a) Developing solar power production in our country (b) Granting licences to foreign T.V. channels in our country (c) Exporting our food products to other countries (d) Permitting foreign educational institutions to set up their campuses in our country Solution: A Why: Frequently in news due to being related to WTO disputes and India’s solar power ambitions. Justification: India‘s Jawaharlal Nehru National Solar Mission (JNNSM) favours Indian manufacturers of solar panels and cells in government procurement. This is known as DCR. = The WTO's Appellate Body has declared domestic content requirement (DCRs) in JNNSM as illegal. = The US alleged that India’s subsidies for the JNNSM tp iwmwinsightsonindia.com/2017/06/1S/answer-koy-upsc-civi-services-pralims-exam-2017-general- studies papor-1/ 40/196 an7017 ‘Answer Key & Detaled Solutions — UPSC Civil Services Prelins Exam -2017 General Studies Paper-1 - INSIGHTS discriminates against foreign suppliers of solar component. The primary point of conflict is with regards to government entering into long term electricity purchase contract with eligible solar power developers (SPDs), assuring them guaranteed prices for 25 years. http://www.downtoearth.org.in/news, rules-against-india-s-domestic- content-requirements-in-solar-power- 50977 28) Consider the following statements: 1. The Nuclear Security Summits are periodically held under the aegis of the United Nations. 2. The International Panel on Fissile Materials is an organ of International Atomic Energy Agency. Which of the statements given above is/are correct? (a) 1 only (b) 2 only tp iwmwinsightsonindia.com/2017/06/1S/answer-koy-upsc-civi-services-pralims-exam-2017-general-stucies-papor-1/ avrias an7017 ‘Answer Key & Detaled Solutions — UPSC Civil Services Prelins Exam -2017 General Studies Paper-1 - INSIGHTS (c) Both 1 and 2 (d) Neither 1 nor 2 Solution: D Justification: Statement 1: It is wrong. In his 2009 speech, Obama (Former US President) stated that nuclear terrorism “is the most immediate and extreme threat to global security.” = To mitigate this threat, he urged that “we act with purpose and without delay,” announcing “a new international effort to secure vulnerable nuclear material around the world” that would begin with “a Global Summit on Nuclear Security that the United States will host.” http://www.nss2016.0rg/about- nss/history/ Statement 2: This is also wrong. « Established in 2006, it is a group of independent nuclear experts from 18 countries: Brazil, Canada, China, France, Germany, India, Iran, Japan, the Netherlands, Mexico, Norway, Pakistan, South Korea, Russia, tp iwmwinsightsonindia.com/2017/06/1S/answer-koy-upsc-civi-services-pralims-exam-2017-general-stucies-papor-1/ a2ra6 ani7a0%7 ‘Answer Key & Detailed Solutions - UPSC Chil Services Prelims Exam -2017 General Studies Paper- 1 - INSIGHTS South Africa, Sweden, the United Kingdom, and the United States. It aims to advance international initiatives to “secure and to sharply reduce all stocks of highly enriched uranium and separated plutonium, the key materials in nuclear weapons, and to limit any further production” www.fissilematerials.org/ipfm/about.h 29. Who among the following can join the National Pension System (NPS)? (a) Resident Indian citizens only (b) Persons of age from 21 to 55 only (c) All State Government employees joining the services after the date of notification by the respective State Governments (d) All Central Government employees including those of Armed Forces joining the services on or after 1St April, 2004 Solution: D tp iwmwinsightsonindia.com/2017/06/1S/answer-koy-upsc-civi-services-pralims-exam-2017-general-stucies-papor-1/ aarta6 ani7a0%7 ‘Answer Key & Detailed Solutions - UPSC Chil Services Prelims Exam -2017 General Studies Paper- 1 - INSIGHTS Justification: Statement 1 and 2: A citizen of India, whether resident or non-resident can join NPS, subject to that he should be between 18 and 60 years old. So, 1 and 2 are wrong. Statement 3 and 4: NPS is applicable to all new employees of Central Government service (except Armed Forces) and Central Autonomous Bodies joining Government service on or after 1st January 2004, So, 3 is wrong and 4 is correct. https://india.gov.in/spotlight/national- pension-system-retirement-plan-all 30) With reference to river Teesta, consider the following statements: 1, The source of river Teesta is the same as that of Brahmaputra but it flows through Sikkim. . River Rangeet originates in Sikkim and it is a tributary of river Teesta. . River Teesta flows into Bay of Bengal on the border of India and Bangladesh. N w Which of the statements given above is/are correct? tp immwinsightsonindia.com/2017/06/19/answer-koy-upsc-civi-services-pralims-exam-2017-general-studies-papor-1/ 4aria6 an7017 Answer Key & Detaled Solutions — UPSC Civil Services Prelims Exam -2017 General Studies Paper 1 - INSIGHTS (a) 1 and 3 only (b) 2 only (©) 2 and 3 only (d) 1, 2 and 3 Solution: B Justification: Statement 1 and 3: Teesta River originates from the Pahunri glacier, Brahmaputra originates in Angsi glacier. Teesta is a tributary of Brahmaputra. So, 1 is wrong. But, Teesta flows through West Bengal and Sikkim, before going to Bay of Bengal through Bangladesh. Statement 3 is incorrect. https://en.wikipedia.org/wiki/Teesta_| http://www.india- wris.nrsc.gov.in/wrpinfo/index.php? title=Brahmaputra Statement 2: Rangeet is a tributary of the Teesta river, which is the largest river in Sikkim. It also originates in Sikkim. https: //en.wikipedia.org/wiki/Rangeet tp iwmwinsightsonindia.com/2017/06/1S/answer-koy-upsc-civi-services-pralims-exam-2017-general-stucies-papor-1/ asii06 an7017 ‘Answer Key & Detaled Solutions — UPSC Civil Services Prelins Exam -2017 General Studies Paper-1 - INSIGHTS 31) Consider the following statements: 1. In tropical regions, Zika virus disease is transmitted by the same mosquito that transmits dengue. 2. Sexual transmission of Zika virus disease is possible. Which of the statements given above is/are correct? (a) 1 only (b) 2 only (c) Both 1 and 2 (d) Neither 1 nor 2 Solution: C tp iwmwinsightsonindia. com/2017/06/1S/answer-koy-upsc-civ-services-prolims-exam-2017-goreral-slusios-paper-t/ 40/196 ani7a0%7 ‘Answer Key & Detailed Solutions - UPSC Chil Services Prelims Exam -2017 General Studies Paper- 1 - INSIGHTS Justification: Statement 1: Zika virus is transmitted to people primarily through the bite of an infected Aedes species mosquito (Ae. aegypti and ‘Ae. albopictus). These are the same mosquitoes that spread dengue and chikungunya viruses, Statement 2: Zika can be passed through sex from a person who has Zika to his or her partners, Zika can be passed through sex, even if the infected person does not have symptoms at the time. https://www.cdc.gov/zika/transmissio 32) Consider the following statements: 1, The Standard Mark of Bureau of Indian Standards (BIS) is mandatory for automotive tyres and tubes. 2, AGMARK is a quality Certification Mark issued by the Food and Agriculture Organisation (FAO). Which of the statements given above is/are correct? (a) 1 only (b) 2 only tp iwmwinsightsonindia.com/2017/06/1S/answer-koy-upsc-civi-services-pralims-exam-2017-general-stucies-papor-1/ a7r36 an7017 Answer Key & Detaled Solutions — UPSC Civil Services Prelims Exam -2017 General Studies Paper 1 - INSIGHTS (c) Both 1 and 2 (d) Neither 1 nor 2 Solution: A Justification: Statement 1: You can find it here http://www.bis.org.in/cert/ProdUnMar Statement 2: AGMARK is a certification mark employed on agricultural products in India, assuring that they conform to a set of standards approved by the Directorate of Marketing and Inspection, an agency of the Government of India. www.agritech.tnau.ac.in/amis/food_a: 33) What is/are the advantage/advantages of implementing the ‘National Agriculture Market’ scheme? 1. It is a pan-India electronic trading portal for agricultural commodities. 2. It provides the farmers access to nationwide market, with prices tp iwmwinsightsonindia.com/2017/06/1S/answer-koy-upsc-civi-services-pralims-exam-2017-general-stucies-papor-1/ 4arta6 an7017 Answer Key & Detaled Solutions — UPSC Civil Services Prelims Exam -2017 General Studies Paper 1 - INSIGHTS commensurate with the quality of their produce. Select the correct answer using the code given below : (a) 1only (b) 2 only (©) Both 1 and 2 (d) Neither 1 nor 2 Solution: C Justification: Statement 1: National Agriculture Market (NAM) is a pan- India electronic trading portal which networks the existing APMC mandis to create a unified national market for agricultural commodities. Statement 2: It removes information asymmetry between buyers and sellers and promotes real time price discovery, based on actual demand and supply, promotes transparency in auction process, and access to a nationwide market for the farmer, with prices commensurate with quality of his produce and online payment and availability of better quality produce and at more reasonable prices to the consumer. http: //www.enam.gov.in/NAM/home/i tp iwmwinsightsonindia.com/2017/06/1S/answer-koy-upsc-civi-services-pralims-exam-2017-general- studies papor-1/ 49/196 an7017 ‘Answer Key & Detaled Solutions — UPSC Civil Services Prelins Exam -2017 General Studies Paper 1 - INSIGHTS 34) With reference to the ‘National Intellectual Property Rights Policy’, consider the following statements: 1. It reiterates India’s commitment to the Doha Development Agenda and the TRIPS Agreement. . Department of Industrial Policy and Promotion is the nodal agency for regulating intellectual property rights in India. N Which of the above statements is/are correct? (a) 1 only (b) 2 only (©) Both 1 and 2 (d) Neither 1 nor 2 Solution: C Justification: Statement 1: This is the reason why this policy was introduced. Statement 2: The action by different Ministries/ Departments shall be monitored by DIPP which shall be the nodal department to coordinate, tp iwmwinsightsonindia.com/2017/06/1S/answer-koy-upsc-civi-services-pralims-exam-2017-general- studies papor-1/ sorts an7017 ‘Answer Key & Detaled Solutions — UPSC Civil Services Prelins Exam -2017 General Studies Paper 1 - INSIGHTS guide and oversee implementation and future development of IPRs in India. http://pib.nic.in/newsite/PrintRelease, relid=145338 35) According to the Wildlife (Protection) Act, 1972, which of the following animals cannot be hunted by any person except under some provisions provided by law? 1. Gharial 2. Indian wild ass 3. Wild buffalo Select the correct answer using the code given below: (a) Lonly (b) 2 and 3 only (c) 1 and 3 only (d) 1, 2 and 3 Solution: D Justification: All of these are protected scheduled animals under the Wildlife Act. http://envfor.nic.in/legis/wildlife/wildli tp iwmwinsightsonindia.com/2017/06/1S/answer-koy-upsc-civi-services-pralims-exam-2017-general-stucies-papor-1/ si/ta6 an7017 ‘Answer Key & Detaled Solutions — UPSC Civil Services Prelins Exam -2017 General Studies Paper-1 - INSIGHTS 36) Which of the following statements is/are true of the Fundamental Duties of an Indian citizen? 1. A legislative process has been provided to enforce these duties. 2. They are correlative to legal duties, Select the correct answer using the code given below: (a) 1 only (b) 2 only (c) Both 1 and 2 (d) Neither 1 nor 2 Solution: C Justification: The language of this question is confusing and ambiguous, nonetheless a meaningful solution can be arrived at. Statement 1: While FDs can be enforced by legislative processes (for e.g. Environment Protection Act that helps enforce the individual duty of environment protection), the tp iwmwinsightsonindia.com/2017/06/1S/answer-koy-upsc-civi-services-pralims-exam-2017-general-stucies-papor-1/ sartae anra017 ‘Answer Key & Detaled Solutions — UPSC Civil Services Prelins Exam -2017 General Studies Paper-1 - INSIGHTS constitution explicitly does not enforce them via legal provisions or does not explicitly mention in Part IVA that there are legislative processes to enforce them. However, since the Parliament has enforced most fundamental duties already via a legislative process, we will consider this statement 1 as correct. Statement 2: It is not clear what UPSC means by ‘legal duties’, but for all practical purposes it can be considered as ‘law of the land’. FDs are in sync with legal duties because what is illegal cannot be a fundamental duty, and what is a fundamental duty has also been made a legal duty (e.g. respecting women). Also, it is an Indian citizen’s fundamental duty to abide by the constitutional laws (legal duties); this brings the correlation. Statement 2 is thus correct. 37) Consider the following pairs: tp iwmwinsightsonindia.com/2017/06/1S/answer-koy-upsc-civi-services-pralims-exam-2017-general-stucies-papor-1/ sates anra017 Answer Key & Detaled Solutions — UPSC Civil Services Prelims Exam -2017 General Studies Paper 1 - INSIGHTS 1. Radhakanta Deb — First President of the British Indian Association 2. Gazulu Lakshminarasu Chetty — Founder of the Madras Mahajana Sabha 3. Surendranath Banerjee — Founder of the Indian Association Which of the above pairs is/are correctly matched? (a) 1 only (b) 1 and 3 only (©) 2 and 3 only (d) 1, 2 and 3 Solution: B Justification: Statement 1: The first committee of the association was composed of: Raja Radhakanta Deb — President, Raja Kalikrishna Deb — Vice-President, Debendranath Tagore — secretary, et al. Statement 2: In 1884, M. Veeraraghavachariar, G. Subramania Iyer and P. Anandacharlu established the Madras Mahajana Sabha. Statement 3: IA was the first avowed nationalist organization founded in British India by tp iwmwinsightsonindia.com/2017/06/1S/answer-koy-upsc-civi-services-pralims-exam-2017-general-stucies-papor-1/ sarta6 an7017 Answer Key & DetoledSolstons -UPSC Chil Sences Prelims Exam 2017 Genera Stes Paper-1 = INSIGHTS Surendranath Banerjee and Ananda Mohan Bose in 1876. https://en.wikipedia.org/wiki/Indian_t https://en.wikipedia.org/wiki/British_I https://en.wikipedia.org/wiki/Madras_ 38. Which one of the following objectives is not embodied in the Preamble to the Constitution of India? (a) Liberty of thought (b) Economic liberty (c) Liberty of expression (d) Liberty of belief Solution: B Straightforward question tp iwmwinsightsonindia.com/2017/06/1S/answer-koy-upsc-civi-services-pralims-exam-2017-general-stucies-papor-1/ ssita6 anra017 ‘Answer Key & Detaled Solutions — UPSC Civil Services Prelins Exam -2017 General Studies Paper 1 - INSIGHTS 39) With reference to ‘Quality Council of India (QCI, consider the following statements: 1. QCI was set up jointly by the Government of India and the Indian Industry. 2. Chairman of QCI is appointed by the Prime Minister on the recommendations of the industry to the Government, Which of the above statements is/are correct? (a) 1only (b) 2 only (c) Both 1 and 2 (d) Neither 1 nor 2 Solution: C Justification: Statement 1: The Quality Council of India (QCI) is a pioneering experiment of the Government of India in setting up organizations in partnership with the Indian industry. Statement 2: QCI is governed by a Council comprising of 38 members including the Chairman and Secretary General where the Chairman is nominated by Prime Minister of India. tp iwmwinsightsonindia.com/2017/06/1S/answer-koy-upsc-civi-services-pralims-exam-2017-general-stucies-papor-1/ se/ta6 an7017 Answer Key & Detaled Solutions — UPSC Civil Services Prelims Exam -2017 General Studies Paper 1 - INSIGHTS http://www.qcin.org/about-qci.php http://www.qcin.org/member.php 40) What is the purpose of setting up of Small Finance Banks (SFBs) in India? 1. To supply credit to small business units 2. To supply credit to small and marginal farmers 3. To encourage young entrepreneurs to set up business particularly in rural areas, Select the correct answer using the code given below: (a) 1 and 2 only (b) 2 and 3 only (c) 1 and 3 only (d) 1,2 and 3 Solution: A Justification: As per RBI Guidelines, “The objectives of setting up of small finance banks will be to further financial inclusion by (a) provision of tp iwmwinsightsonindia.com/2017/06/1S/answer-koy-upsc-civi-services-pralims-exam-2017-general-stucies-papor-1/ sritae an7017 Answer Key & Detaled Solutions — UPSC Civil Services Prelims Exam -2017 General Studies Paper 1 - INSIGHTS savings vehicles, and (ii) supply of credit to small business units; small and marginal farmers; micro and small industries; and other unorganised sector entities, through high technology-low cost operations.” It will thus support unserved and underserved sections. However, apart from the provision of credit, there is no explicit provision that SFBs have been setup to encourage setting of businesses in rural areas. Statement 3 will be considered wrong. https://www.rbi.org.in/Scripts/BS_Pre prid=32614 http://www.livemint.com/Money/LICg and-loans-with-the-new-small- banks.html 41) With reference to ‘Asia Pacific Ministerial Conference on Housing and Urban Development (APMCHUDY’, consider the following statements: tp iwmwinsightsonindia.com/2017/06/1S/answer-koy-upsc-civi-services-pralims-exam-2017-general- studies papor-1/ sertae ani7a0%7 ‘Answer Key & Detailed Solutions - UPSC Chil Services Prelims Exam -2017 General Studies Paper 1 - INSIGHTS 1. The first APMCHUD was held in India in 2006 on the theme ‘Emerging Urban Forms — Policy Responses and Governance Structure’. . India hosts all the Annual Ministerial Conferences in partnership with ADB, APEC and ASEAN. N Which of the statements given above is/are correct? (a) 1only (b) 2 only (©) Both 1 and 2 (d) Neither 1 nor 2 Solution: D Justification: Statement 1: The 2006 New Delhi APMCHUD conference theme was “A vision for sustainable urbanisation in the Asia-Pacific by 2020”. So, 1 is wrong. Statement 2: Second Conference was held in Tehran, Iran. http://www.apmchud.com/PDF/1st%: http://www.apmchud.com/2stConfere tp iwmwinsightsonindia.com/2017/06/1S/answer-koy-upsc-civi-services-pralims-exam-2017-general-stucies-papor-1/ sartae anra017 Answer Key & Detaled Solutions — UPSC Civil Services Prelims Exam -2017 General Studies Paper 1 - INSIGHTS 42. Democracy’s superior virtue lies in the fact that it calls into activity (a) the intelligence and character of ordinary men and women. (b) the methods for strengthening executive leadership. (c) a superior individual with dynamism and vision. (d) a band of dedicated party workers, Solution: A Justification: This is another ambiguously framed question. Option A: Democracy allows you to vote (by exercising your intellect), it gives you liberty and freedom of expression (so that you intellectual potential is fulfilled), and ensures adherence of moral values (character) such as justice and equality. So, A seems appropriate. Option B: There is no ground for this statement. Option C: Since it specifies a “superior individual”, it attaches a sense of privilege to selected people, tp iwmwinsightsonindia.com/2017/06/1S/answer-koy-upsc-civi-services-pralims-exam-2017-general-stucies-papor-1/ sortase anra017 ‘Answer Key & Detaled Solutions - UPSC Givi Series Palins Exam -2017 General Stsies Paper~1 - INSIGHTS which is against democratic values. It also does not clarify what the dynamism and vision means. Option D: No ground. 43. Which of the following is a most likely consequence of implementing the ‘Unified Payments Interface (UPI)’? (a) Mobile wallets will not be necessary for online payments. (b) Digital currency will totally replace the physical currency in about two decades. (c) FDI inflows will drastically increase, (d) Direct transfer of subsidies to poor people will become very effective. Solution: A Justification: Option A and D: UPI allows a customer to pay directly from a bank account to different merchants, both online and offline, tp iwmwinsightsonindia.com/2017/06/1S/answer-koy-upsc-civi-services-pralims-exam-2017-general- studies papor-1/ srrta6 an7017 ‘Answer Key & Detaled Solutions - UPSC Givi Series Palins Exam -2017 General Stsies Paper~1 - INSIGHTS without the hassle of typing credit card details, IFSC code, or net banking/wallet passwords. So, A is correct and thus D is inappropriate in this context (as we are not talking about government to people transfer through UPI). Option B and C: These are uncertain events, hence discarded. http://www.thehindu.com/business/Ei is-Unified-Payment- Interface/article14593189.ece 44. The terms ‘Event Horizon’, ‘Singularity’, “String Theory’ and ‘Standard Model’ are sometimes seen in the news in the context of (a) Observation and understanding of the Universe (b) Study of the solar and the lunar eclipses (c) Placing satellites in the orbit of the Earth (d) Origin and evolution of living organisms on the Earth tp iwmwinsightsonindia.com/2017/06/1S/answer-koy-upsc-civi-services-pralims-exam-2017-general-stucies-papor-1/ ears an7017 Answer Key & Detaled Solutions — UPSC Civil Services Prelims Exam -2017 General Studies Paper 1 - INSIGHTS Solution: A Justification: Singularity and event horizons are related to Black Holes. Standard model of physics tries to explain universal phenomena. String theory is used in the context of quantum physics that is used to understand quantum phenomena. 45) With reference to agriculture in India, how can the technique of * genome sequencing’, often seen in the news, be used in the immediate future? 1. Genome sequencing can be used to identify genetic markers for disease resistance and drought tolerance in various crop plants. This technique helps in reducing the time required to develop new varieties of crop plants. . It can be used to decipher the host-pathogen relationships in crops. N w Select the correct answer using the code given below: tp iwmwinsightsonindia.com/2017/06/1S/answer-koy-upsc-civi-services-pralims-exam-2017-general- studies papor-1/ sartas an7017 ‘Answer Key & Detaled Solutions — UPSC Civil Services Prelins Exam -2017 General Studies Paper 1 - INSIGHTS (a) 1 only (b) 2 and 3 only (©) 1 and 3 only (d) 1, 2 and 3 Solution: D Justification: Genome sequencing is figuring out the order of DNA nucleotides, or bases, in a genome— the order of As, Cs, Gs, and Ts that make up an organism's DNA. Justification: Statement 1: Currently available newborn screening (genome) for childhood diseases allows detection of rare disorders that can be prevented or better treated by early detection and intervention. https://www.scientificamerican.com/a genome-sequencing-predict-disease/ Statement 2: Naturally if the gene potential can be identified by screening and sequencing, it will help in better genetic engineering. Statement 3: This can be reasoned logically based on the above. https: //en.wikipedia.org/wiki/Whole_c tp iwmwinsightsonindia.com/2017/06/1S/answer-koy-upsc-civi-services-pralims-exam-2017-general-stucies-papor-1/ oarta6 anra017 Answer Key & Detaled Solutions — UPSC Civil Services Prelims Exam -2017 General Studies Paper 1 - INSIGHTS 46. The main advantage of the parliamentary form of government is that (a) the executive and legislature work independently. (b) it provides continuity of policy and is more efficient. (©) the executive remains responsible to the legislature. (d) the head of the government cannot be changed without election. Solution: C Straightforward question based on the core Parliamentary democracy principles. 47. In the context of India, which one of the following is the correct relationship between Rights and Duties? (a) Rights are correlative with Duties. tp iwmwinsightsonindia.com/2017/06/1S/answer-koy-upsc-civi-services-pralims-exam-2017-general-stucies-papor-1/ esita6 anra017 tp iwmwinsightsonindia.com/2017/06/1S/answer-koy-upsc-civi-services-pralims-exam-2017-general-stucies-papor-1/ ‘Answer Key & Detaled Solutions — UPSC Civil Services Prelins Exam -2017 General Studies Paper-1 - INSIGHTS (b) Rights are personal and hence independent of society and Duties. (© Rights, not Duties, are important for the advancement of the personality of the citizen. (d) Duties, not Rights, are important for the stability of the State. Solution: A Justification: Rights and duties have an organic relationship. One cannot enjoy a right if others do not obey their duties. For e.g. you can’t enjoy your Right to a clean environment if others do not perform their duty of protecting and conserving the environment. 48. The mind of the makers of the Constitution of India is reflected in which of the following? (a) The Preamble (b) The Fundamental Rights (c) The Directive Principles of State Policy ssrtas anra017 Answer Key & Detaled Solutions — UPSC Civil Services Prelims Exam -2017 General Studies Paper 1 - INSIGHTS (d) The Fundamental Duties Solution: A Straightforward question 49. If you travel by road from Kohima to Kottayam, what is the minimum number of States within India through which you can travel, including the origin and the destination? (a)6 (b)7 (8 (9 Solution: B Justification: You need to cross Nagaland (origin), Assam, WB, Odisha, Andhra Pradesh and then through either Tamilnadu to Kerala (Kottayam), or through AP to Karnataka to Kerala: 7 states either way. tp iwmwinsightsonindia.com/2017/06/1S/answer-koy-upsc-civi-services-pralims-exam-2017-general-stucies-papor-1/ erita6 an7017 ‘Answer Key & Detaled Solutions — UPSC Civil Services Prelins Exam -2017 General Studies Paper-1 - INSIGHTS 50) The Parliament of India exercises control over the functions of the Council of Ministers through 1. Adjournment motion 2. Question hour 3. Supplementary questions Select the correct answer using the code given below: (a) 1 only (b) 2 and 3 only (c) 1 and 3 only (d) 1, 2. and 3 Solution: D Justification: Statement 1: Matters of urgent importance can be raised in tp iwmwinsightsonindia. com/2017/06/1S/answer-koy-upsc-civ-services-prolims-exam-2017-goreral-slusios-paper-t/ oarta6 an7017 Answer Key & Detaled Solutions — UPSC Civil Services Prelims Exam -2017 General Studies Paper 1 - INSIGHTS Adjournment motion. Statement 2 and 3: MPs hold the executive accountable by asking questions, and supplementary questions. 51. Which one of the following was a very important seaport in the Kakatiya kingdom? (a) Kakinada (b) Motupalli (c) Machilipatnam (Masulipatnam) (d) Nelluru Solution: B Justification: There is dispute whether the answer should be B, C or D. Option B: Some texts (http://tinyurl.com/yby78f8d) suggest that Motupalli, now in the Krishna district, was an important sea-port in the kingdom of Ganpati (an important Kakatiya ruler), frequented by foreign merchants. Option C: Marco Polo, who visited India probably some time around tp iwmwinsightsonindia.com/2017/06/1S/answer-koy-upsc-civi-services-pralims-exam-2017-general-stucies-papor-1/ 9/96 an7017 ‘Answer Key & Detaled Solutions - UPSC Gil Series Palins Exam -2017 General Stsies Paper-1 - INSIGHTS 1289-1293, made note of Rudrama Devi’s (a Kakatiya ruler) rule and nature in flattering terms. Marco Polo referred to the kingdom as Mutfili, which was the name for the area around a major port of the dynasty, now known as Masulipatnam. But, Marco Polo's reference may actually be to Motupalli (Mutfilli). So, Cis incorrect, http://tinyurl.com/y8y3wb9v https: //en.wikipedia.org/wiki/Kakatiya 26 Option D: Nellore (a port town) was part of kakatiya kingdom in the 13th century. It changed hands between them and Pandyas few times until Prataprudra II defeated Pandyas. However, this does not seem as important as Motupalli. Answer will be B. https://en.wikipedia.org/wiki/Nellore_ 52) With reference to ‘Global Climate Change Alliance’, which of the tp iwmwinsightsonindia.com/2017/06/1S/answer-koy-upsc-civi-services-pralims-exam-2017-general-stucies-papor-1/ Torta6 an7017 Answer Key & Detaled Solutions — UPSC Civil Services Prelims Exam -2017 General Studies Paper 1 - INSIGHTS following statements is/are correct? 1. It is an initiative of the European Union, . It provides technical and financial support to targeted developing countries to integrate climate change into their development policies and budgets. . It is coordinated by World Resources Institute (WRI) and World Business Council for Sustainable Development (WBCSD). w Select the correct answer using the code given below: (a) 1 and 2 only (b) 3 only (c) 2 and 3 only (d) 1, 2 and 3 Solution: A Statement 1: The GCCA was established by the European Union (EU) in 2007 to strengthen dialogue and cooperation with developing countries, in particular least developed countries (LDCs) and small island developing States (SIDS). Statement 2: By fostering effective dialogue and cooperation on climate tp iwmwinsightsonindia.com/2017/06/1S/answer-koy-upsc-civi-services-pralims-exam-2017-general-stucies-papor-1/ Tta6 ani7a0%7 ‘Answer Key & Detailed Solutions - UPSC Chil Services Prelims Exam -2017 General Studies Paper 1 - INSIGHTS change, the Alliance helps to ensure that poor developing countries most vulnerable to climate change increase their capacities to adapt to the effects of climate change, in support of the achievement of the Millenium Development Goals (MDGs). Statement 3: There is no mention of WRI and WBCSD at the official partners page of GCCA, even though a lot of other institutions are mentioned like FAO, UNDP etc. http://www.gcca.eu/about-the- gcca/partners 53) With reference to the religious history of India, consider the following statements: 1. Sautrantika and Sammitiya were the sects of Jainism. 2. Sarvastivadin held that the constituents of phenomena were not wholly momentary, but existed forever in a latent form. Which of the statements given above is/are correct? (a) 1 only (b) 2 only tp iwmwinsightsonindia.com/2017/06/1S/answer-koy-upsc-civi-services-pralims-exam-2017-general-stucies-papor-1/ 7aita6 an7017 tpn ‘Answer Key & Detaled Solutions — UPSC Civil Services Prelins Exam -2017 General Studies Paper 1 - INSIGHTS (c) Both 1 and 2 (d) Neither 1 nor 2 Solution: B Justification: Statement 1: These are sects of Buddhism. Statement 2: While, like all Buddhists, the Sarvastivadins consider everything empirical to be impermanent, they maintain that the dharma factors are eternally existing realities. The dharmas are thought to function momentarily, producing the empirical phenomena of the world, which is illusory, but to exist outside the empirical world. https://www.britannica.com/topic/Sar 54) Mediterranean Sea is a border of which of the following countries? 1. Jordan 2. Iraq 3. Lebanon 4, Syria Select the correct answer using the code given below: insightsonindia. com2017/06/19/answer-koy-upsc-civi-services-pralims-exam-2017-general-studies-paper-1/ 7aita6 an7017 Answer Key & Detaled Solutions — UPSC Civil Services Prelims Exam -2017 General Studies Paper 1 - INSIGHTS (a) 1, 2 and 3 only (b) 2 and 3 only (©) 3 and 4 only (d) 1, 3 and 4 only Solution: C Justification: 55) With reference to ‘National Investment and Infrastructure Fund’, which of the following statements is/are correct? 1. It is an organ of NITI Aayog. 2. It has a corpus of Rs. 4, 00,000 crore at present. Select the correct answer using the code given below: tp iwmwinsightsonindia. com/2017/06/1S/answer-koy-upsc-civ-services-prolims-exam-2017-goreral-slusios-paper-t/ rariae an7017 ‘Answer Key & Detaled Solutions — UPSC Civil Services Prelins Exam -2017 General Studies Paper-1 - INSIGHTS (a) 1 only (b) 2 only (©) Both 1 and 2 (d) Neither 1 nor 2 Solution: D Justification: NIIF has been structured as a fund of funds and set up as Category II Alternate Investment Fund (AIF) under the Securities and Exchange Board of India (SEBI) Regulations. Total corpus of the fund is Rs. 40000 Crore. http://arthapedia.in/index.php%3Ftith 56. The Global Infrastructure Facility is a/an (a) ASEAN initiative to upgrade infrastructure in Asia and financed by credit from the Asian Development Bank. (b) World Bank collaboration that facilitates the preparation and structuring of complex infrastructure Public-Private Partnerships (PPPs) to tp iwmwinsightsonindia.com/2017/06/1S/answer-koy-upsc-civi-services-pralims-exam-2017-general- studies papor-1/ 7sit96 an7017 ‘Answer Key & Detaled Solutions — UPSC Civil Services Prelins Exam -2017 General Studies Paper 1 - INSIGHTS enable mobilization of private sector and institutional investor capital. (©) Collaboration among the major banks of the world working with the OECD and focused on expanding the set of infrastructure projects that have the potential to mobilize private investment. (d) UNCTAD funded initiative that seeks to finance and facilitate infrastructure development in the world. Solution: B Justification: It was launched by the World Bank to cater to infrastructure needs of countries. http://www.globalinfrafacility.org/whe is-the-gif 57. For election to the Lok Sabha, a nomination paper can be filed by (a) anyone residing in India. (b) a resident of the constituency from which the election is to be contested. tp iwmwinsightsonindia.com/2017/06/1S/answer-koy-upsc-civi-services-pralims-exam-2017-general- studies papor-1/ Teita6 anra017 tp iwmwinsightsonindia.com/2017/06/1S/answer-koy-upsc-civi-services-pralims-exam-2017-general-stucies-papor-1/ ‘Answer Key & Detaled Solutions — UPSC Civil Services Prelins Exam -2017 General Studies Paper 1 - INSIGHTS (©) any citizen of India whose name appears in the electoral roll of a constituency. (d) any citizen of India. Solution: C Straightforward question. 58) Consider the following statements: . In India, the Himalayas are spread over five States only. . Western Ghats are spread over five States only. . Pulicat Lake is spread over two States only. w Which of the statements given above is/are correct? (a) 1 and 2 only (b) 3 only (c) 2 and 3 only (d) 1 and 3 only Solution: B Statement 1: Himalayas are spread over almost all the Northern and Tita6 anra017 ‘Answer Key & Detaled Solutions — UPSC Civil Services Prelins Exam -2017 General Studies Paper-1 - INSIGHTS north-eastern Indian states. Statement 1 is wrong. Statement 2: Western Ghats traverse the States of Kerala, Tamil Nadu, Karnataka, Goa, Maharashtra and Gujarat (6 states). Statement 3: Pulicat Lake (second largest brackish water lake in India)straddles the border of Andhra Pradesh and Tamil Nadu states (2 states). 59. Biological Oxygen Demand (BOD) is a standard criterion for (a) Measuring oxygen levels in blood (b) Computing oxygen levels in forest ecosystems (c) Pollution assay in aquatic ecosystems (d) Assessing oxygen levels in high altitude regions Solution: C Justification: It is the amount of dissolved oxygen needed (i.e., demanded) by aerobic biological tp iwmwinsightsonindia.com/2017/06/1S/answer-koy-upsc-civi-services-pralims-exam-2017-general-stucies-papor-1/ Taita6 an7017 ‘Answer Key & Detaled Solutions — UPSC Civil Services Prelins Exam -2017 General Studies Paper 1 - INSIGHTS organisms to break down organic material present in a given water sample at certain temperature over a specific time period. https: //en.wikipedia.org/wiki/Biochem 60) With reference to the role of UN- Habitat in the United Nations programme working towards a better urban future, which of the statements is/are correct? 1. UN-Habitat has been mandated by the United Nations General Assembly to promote socially and environmentally sustainable towns and cities to provide adequate shelter for all. . Its partners are either governments or local urban authorities only. . UN-Habitat contributes to the overall objective of the United Nations system to reduce poverty and to promote access to safe drinking water and basic sanitation. N w tp iwmwinsightsonindia.com/2017/06/1S/answer-koy-upsc-civi-services-pralims-exam-2017-general-stucies-papor-1/ rarta6 an7017 ‘Answer Key & Detaled Solutions — UPSC Civil Services Prelins Exam -2017 General Studies Paper 1 - INSIGHTS Select the correct answer using the code given below: (a) 1, 2and3 (b) 1 and 3 only (c) 2 and 3 only (d) 1 only Solution: B Why: India was recently (March 2017) unanimously elected as the President of Un-Habitat. http://pib-nic.in/newsite/PrintRelease. relid=161631 Justification: Statement 1 and 3; They are correct. https://unhabitat.org/about- us/history-mandate-role-in-the-un- system/ Statement 2: Through drawing together cooperation with committed partners, relevant stakeholders, and urban actors, including at all levels of government as well as the private sector, UN-Habitat is applying its technical expertise to the New Urban Agenda and Sustainable Development Goa tp iwmwinsightsonindia.com/2017/06/1S/answer-koy-upsc-civi-services-pralims-exam-2017-general-stucies-papor-1/ 0/196 an7017 ‘Answer Key & Detaled Solutions — UPSC Civil Services Prelins Exam -2017 General Studies Paper 1 - INSIGHTS 61) With reference to ‘National Skills Qualification Framework (NSQF)’, which of the statements given below is/are correct? 1. Under NSQF, a learner can acquire the certification for competency only through formal learning. . An outcome expected from the implementation of NSQF is the mobility between vocational and general education. N Select the correct answer using the code given below: (a) 1 only (b) 2 only (©) Both 1 and 2 (d) Neither 1 nor 2 Solution: B Why: Schemes of the Ministries and Departments lifted from their official websites. Justification: Statement 1: NSQF levels, graded from one to ten, are defined in terms of learning ‘outcomes which the learner must. tp iwmwinsightsonindia.com/2017/06/1S/answer-koy-upsc-civi-services-pralims-exam-2017-general-stucies-papor-1/ arrta6 an7017 ‘Answer Key & Detaled Solutions — UPSC Civil Services Prelins Exam -2017 General Studies Paper 1 - INSIGHTS possess regardless of whether they are obtained through formal, non- formal or informal learning. Statement 2: Specific outcomes expected from implementation of NSQF are: = Mobility between vocational and general education by alignment of degrees with NSQF Recognition of Prior Learning (RPL), allowing transition from non-formal to organised job market Standardised, consistent, nationally acceptable outcomes of training across the country through a national quality assurance framework http: //www.skilldevelopment.gov.in/n 62. In the context of Indian history, the principle of * Dyarchy (diarchy)’ refers to (a) Division of the central legislature into two houses. (b) Introduction of double government i.e., Central and State tp iwmwinsightsonindia.com/2017/06/1S/answer-koy-upsc-civi-services-pralims-exam-2017-general- studies papor-1/ a2ria6 anra017 ‘Answer Key & Detaled Solutions — UPSC Civil Services Prelins Exam -2017 General Studies Paper-1 - INSIGHTS governments. (c) Having two sets of rulers; one in London and another in Delhi. (d) Division of the subjects delegated to the provinces into two categories. Solution: D Straightforward question 63) Consider the following in respect of ‘National Career Service’: 1, National Career Service is an initiative of the Department of Personnel and Training, Government of India. National Career Service has been launched in a Mission Mode to improve the employment opportunities to uneducated youth of the country. Which of the above statements is/are correct? (a) 1 only (b) 2 only (c) Both 1 and 2 tp iwmwinsightsonindia.com/2017/06/1S/answer-koy-upsc-civi-services-pralims-exam-2017-general-stucies-papor-1/ eartas an7017 Answer Key & Detaled Solutions — UPSC Civil Services Prelims Exam -2017 General Studies Paper 1 - INSIGHTS (d) Neither 1 nor 2 Solution: B Why: Schemes of the Ministries and Departments lifted from their official websites. Justification: Statement 1: It is an initiative launched by the Ministry of Labour and Employment. Statement 2: It is a National ICT based portal developed primarily to connect the opportunities with the aspirations of youth. This portal facilitates registration of job seekers, job providers, skill providers, career counsellors, etc. http://www. labour.nic.in/national- career-servicencs 64. Which of the following statements best describes the term ‘Scheme for Sustainable Structuring of Stressed Assets (S4Ay’, recently seen in the news? tp iwmwinsightsonindia.com/2017/06/1S/answer-koy-upsc-civi-services-pralims-exam-2017-general-stucies-papor-1/ aria6 anra017 ‘Answer Key & Detaled Solutions — UPSC Civil Services Prelins Exam -2017 General Studies Paper-1 - INSIGHTS (a) It is a procedure for considering ecological costs of developmental schemes formulated by the Government. (b) It is a scheme of RBI for reworking the financial structure of big corporate entities facing genuine difficulties. (0 It isa disinvestment plan of the Government regarding Central Public Sector Undertakings. (d) It is an important provision in ‘The Insolvency and Bankruptcy Code’ recently implemented by the Government. Solution: B Why: Either one of Joint Lender's Forum (JLF) or S4A was expected due to the ongoing issue of large scale NPAs in the banking sector. Justification: The Reserve Bank of India (RBI) has issued guidelines called Scheme for Sustainable Structuring of Stressed Assets (S4A) as an optional framework to manage bad loans. Under this scheme, large ticket loans are restructured by separating a sustainable loan from an unsustainable loan. tp iwmwinsightsonindia.com/2017/06/1S/answer-koy-upsc-civi-services-pralims-exam-2017-general- studies papor-1/ asita6 ani7a0%7 ‘Answer Key & Detailed Solutions - UPSC Chil Services Prelims Exam -2017 General Studies Paper- 1 - INSIGHTS The lenders are required to make this classification. Sustainable level of debt is one which the banks think the stressed borrower can service with its current cash flows. https://www.rbi.org.in/scripts/Notifica 1d=104468Mode=0 65) Consider the following statements: 1. Climate and Clean Air Coalition (CCAC) to Reduce Short Lived Climate Pollutants is a unique initiative of G20 group of countries. 2. The CCAC focuses on methane, black carbon and hydrofluorocarbons. Which of the statements given above is/are correct? (a) 1 only (b) 2 only (©) Both 1 and 2 (d) Neither 1 nor 2 tp iwmwinsightsonindia.com/2017/06/1S/answer-koy-upsc-civi-services-pralims-exam-2017-general-stucies-papor-1/ 6/196 an7017 ‘Answer Key & Detaled Solutions — UPSC Civil Services Prelins Exam -2017 General Studies Paper 1 - INSIGHTS Solution: B Justification: Statement 1: The Climate and Clean Air Coalition to Reduce Short-Lived Climate Pollutants (CCAC) was launched by the United Nations Environment Programme (UNEP) and six countries —Bangladesh, Canada, Ghana, Mexico, Sweden, and the United States—in 2012, Statement 2: The Coalition's initial focus is on methane, black carbon, and HFCs. http://www.ccacoalition.org/en/conte! us 66) With reference to ‘Indian Ocean Dipole (IOD)’ sometimes mentioned in the news while forecasting Indian monsoon, which of the following statements is/are correct? 1. IOD phenomenon is characterised by a difference in sea surface temperature between tropical Western Indian Ocean and tropical Eastern Pacific Ocean. tp iwmwinsightsonindia.com/2017/06/1S/answer-koy-upsc-civi-services-pralims-exam-2017-general- studies papor-1/ arria6 anra017 Answer Key & Detaled Solutions — UPSC Civil Services Prelims Exam -2017 General Studies Paper 1 - INSIGHTS 2. An IOD phenomenon can influence an El Nino's impact on the monsoon, Select the correct answer using the code given below: (a) Lonly (b) 2 only (c) Both 1 and 2 (d) Neither 1 nor 2 Solution: B Why: The IOD is frequently seen in news before the onslaught on Monsoon in India, IMD bases its Monsoon predictions on both IOD and El-Nino. This was an expected question. Justification: Statement 1: The IOD, also known as the Indian Nifio, is an irregular oscillation of sea-surface temperatures in which the western Indian Ocean becomes alternately warmer and then colder than the eastern part of the Indian ocean (not tropical eastern pacific ocean. Statement 2: IOD has a much more significant effect on the rainfall patterns in south-east Australia than the El Nifio-Southern Oscillation tp iwmwinsightsonindia.com/2017/06/1S/answer-koy-upsc-civi-services-pralims-exam-2017-general- studies papor-1/ eerta6 an7017 Answer Key & DetoledSolstons -UPSC Chil Sences Prelims Exam 2017 General Stes Paper-1 = INSIGHTS (ENSO) in the Pacific Ocean as shown in several recent studies. http://www.bom.gov.au/climate/iod/ https://en.wikipedia.org/wiki/Indian_¢ 67. If you want to see gharials in their natural habitat, which one of the following is the best place to visit? (a) Bhitarkanika Mangroves (b) Chambal River (c) Pulicat Lake (d) Deepor Beel Solution: B Why: A critically endangered species especially featured in WWF India website: http://www.wwfindia.org/indian_ghar Justification: Option A: Gharials inhabit deep freshwater habitats, not both freshwater and saltwater habitats as inhabited by crocodile species. So, Ais ruled out. Option A is the home of saltwater crocodile. tp iwmwinsightsonindia.com/2017/06/1S/answer-koy-upsc-civi-services-pralims-exam-2017-general-stucies-papor-1/ 9/196 avi7e017 ‘Answer Key & Detaled Solutions - UPSC Givi Series Palins Exam -2017 General Stsies Paper~1 - INSIGHTS Option B: Gharials can be naturally found in National Chambal Gharial Wildlife Sanctuary. The gharial is one of three crocodilians native to India, the other two being the mugger crocodile and the saltwater crocodile, 68) Consider the following in respect of Indian Ocean Naval Symposium (IONS): 1. Inaugural IONS was held in India in 2015 under the chairmanship of the Indian Navy. . IONS is a voluntary initiative that N seeks to increase maritime co- operation among navies of the littoral states of the Indian Ocean Region. Which of the above statements is/are correct? (a) t only (b) 2 only (©) Both 1 and 2 (d) Neither 1 nor 2 tp iwmwinsightsonindia.com/2017/06/1S/answer-koy-upsc-civi-services-pralims-exam-2017-general-stucies-papor-1/ sorta an7017 ‘Answer Key & Detaled Solutions — UPSC Civil Services Prelins Exam -2017 General Studies Paper-1 - INSIGHTS Solution: B Justification: Statement 1: The inaugural IONS-2008 was held in New Delhi, India. CNS, Indian Navy was designated the Chairman IONS for the period 2008-10. Statement 2: It is correct http://ions.gov.in/about_ions 69. The painting of Bodhisattva Padmapani is one of the most famous and oft-illustrated paintings at (a) Ajanta (b) Badami (©) Bagh (d) Ellora Solution: A Standard question from NCERT books tp iwmwinsightsonindia.com/2017/06/1S/answer-koy-upsc-civi-services-pralims-exam-2017-general-stucies-papor-1/ ovis an7017 ‘Answer Key & Detaled Solutions — UPSC Civil Services Prelins Exam -2017 General Studies Paper 1 - INSIGHTS 70. Consider the following pairs: Traditions Commu 1. Chaliha Sahib Festival - Sindhis 2. Nanda Raj Jaat Yatra _ Gonds 3. Wari-Warkari Santhals Which of the pairs given above is/are correctly matched ? (a) 1 only (b) 2 and 3 only (c) 1 and 3 only (d) None of the above Solution: A Justification: Statement 1: www.chalihasahib.com Statement 2: Nanda Devi Raj Jaat Yatra is a festival of Gharwali and kumauoni people in Uttarakhand. Statement 3: Wari Warkari or Varkari (Pandharpur Wari) is a Maharashtrian festival celebrated by Marathas, tp iwmwinsightsonindia.com/2017/06/1S/answer-koy-upsc-civi-services-pralims-exam-2017-general-stucies-papor-1/ s2rta6 an7017 ‘Answer Key & Detaled Solutions — UPSC Civil Services Prelins Exam -2017 General Studies Paper 1 - INSIGHTS 71. Which of the following practices can help in water conservation in agriculture? 1, Reduced or zero tillage of the land 2. Applying gypsum before irrigating the field 3. Allowing crop residue to remain in the field Select the correct answer using the code given below : (a) 1 and 2 only (b) 3 only (©) 1 and 3 only (d) 1, 2 and 3 Solution: D Why: Standard questions that are lifted from www.agritech.tnau.ac.in every year. Expect 2-3 questions on core agricultural practices next year as well. Statement 1: Tillage is the agricultural preparation of soil by mechanical agitation of various tp iwmwinsightsonindia.com/2017/06/1S/answer-koy-upsc-civi-services-pralims-exam-2017-general-stucies-papor-1/ saree anra017 tpi Answer Key & Detaled Solutions UPSC Cl Senices Prelim Exam 2017 General Suis Paper-1 - INSIGHTS types, such as digging, stirring, and overturning. No-till farming (also called zero tillage or direct drilling) is a way of growing crops or pasture from year to year without disturbing the soil through tillage. Zero tillage helps in soil moisture conservation. Statement 2: An infiltration problem often occurs in farms if the irrigation water does not enter the soil rapidly enough during a normal irrigation cycle. Gypsum helps in dissolution of the salt clay pan found in soils that hinder the percolation of water. This increases the efficiency of irrigation as less water is needed to irrigate the same soil. Adding gypsum to irrigation water further increases its infiltration rate. Statement 3: This reduces evaporation of farm soil water. 72. Consider the following statements: insightsonindia. com2017/06/19/answer-koy-upsc-civi-services-pralims-exam-2017-general-studies-paper-1/ artes an7017 ‘Answer Key & Detaled Solutions — UPSC Civil Services Prelins Exam -2017 General Studies Paper-1 - INSIGHTS The nation-wide ‘Soil Health Card Scheme’ aims at 1. expanding the cultivable area under irrigation. . enabling the banks to assess the quantum of loans to be granted to farmers on the basis of soil quality. . checking the overuse of fertilizers in farmlands, N w Which of the above statements is/are correct? (a) 1 and 2 only (b) 3 only (c) 2 and 3 only (d) 1, 2 and 3 Solution: B Justification: Statement 3: Soil Health card (SHC) is a printed report card issued to farmers in once in three years indicating the status of his soil in terms of 12 parameters. It is also accompanied by an advice on the various fertilizers and other soil amendments he is suppose to make. So, only 3 is correct. Statement 2: This can be an outcome of the scheme and complement it, tp iwmwinsightsonindia.com/2017/06/1S/answer-koy-upsc-civi-services-pralims-exam-2017-general-stucies-papor-1/ 9sita6 an7017 ‘Answer Key & Detaled Solutions — UPSC Civil Services Prelins Exam -2017 General Studies Paper 1 - INSIGHTS but it is not an aim or a central feature of the scheme. http://www.arthapedia.in/index.php? title=Soil_Health_Card_(SHC) 73. Consider the following pairs: Commonly used / consumed materials and Unwanted or controversial chemicals likely to be found in them 1. Lipstick 2. Soft drinks Br vegetable oils 3. Chinese fast food Monosodiu glutamate Which of the pairs given above is/are correctly matched? (a) 1 only (b) 2 and 3 only (© Land 3 only (d) 1, 2and3 tp iwmwinsightsonindia.com/2017/06/1S/answer-koy-upsc-civi-services-pralims-exam-2017-general-stucies-papor-1/ 6196 an7017 ‘Answer Key & Detaled Solutions — UPSC Civil Services Prelins Exam -2017 General Studies Paper 1 - INSIGHTS Solution: D Why: The Maggi MSG Episode and the opposition of cosmetic animal testing by several NGOs recently Justification: Statement 1: It is one of the ingredients. www.safecosmetics.org/get-the- facts/regulations/us-laws/lead-in- lipstick/ Statement 2: Brominated vegetable oil is used primarily to help emulsify citrus-flavored soft drinks, preventing them from separating during distribution. Statement 3: This was found in Maggi. 74) Organic Light Emitting Diodes (OLEDs) are used to create digital display in many devices. What are the advantages of OLED displays over Liquid Crystal displays? 1. OLED displays can be fabricated on flexible plastic substrates. 2, Roll-up displays embedded in clothing can be made using OLEDs. tp iwmwinsightsonindia.com/2017/06/1S/answer-koy-upsc-civi-services-pralims-exam-2017-general-stucies-papor-1/ o7ra6 anra017 Answer Key & Detaled Solutions UPSC Cl Senices Prelim Exam 2017 General Suis Paper-1 - INSIGHTS 3. Transparent displays are possible using OLEDs. Select the correct answer using the code given below (a) 1 and 3 only (b) 2 only (©) 1, 2and3 (d) None of the above statements is correct Solution: C Why: Introduction of OLED screens in TVs and smartphones by major brands: The question statements were directly lifted from Wikipedia. Justification: Statement 1 and 2: A major advantage of OLED over LCD is that it is lightweight and has flexible plastic substrates. = OLED displays can be fabricated ‘on flexible plastic substrates, leading to the possible fabrication of flexible organic light-emitting diodes for other new applications, such as roll-up displays embedded in fabrics or clothing. If a substrate like polyethylene terephthalate (PET) can be used, the displays may be produced inexpensively. tp iwmwinsightsonindia.com/2017/06/1S/answer-koy-upsc-civi-services-pralims-exam-2017-general-stucies-papor-1/ serta6 an7017 ‘Answer Key & Detaled Solutions - UPSC Givi Series Palins Exam -2017 General Stsies Paper~1 - INSIGHTS = Furthermore, plastic substrates are shatter-resistant, unlike the glass displays used in LCD devices. Statement 3: At the Consumer Electronics Show (CES) in 2010, Samsung demonstrated a laptop computer with a large, transparent OLED display featuring up to 40% transparency. 75) Which of the following is/are famous for Sun temples? 1. Arasavalli 2. Amarakantak 3. Omkareshwar Select the correct answer using the code given below : (a) 1 only (b) 2 and 3 only (c) 1 and 3 only (d) 1, 2. and 3 Solution: A tp iwmwinsightsonindia.com/2017/06/1S/answer-koy-upsc-civi-services-pralims-exam-2017-general-stucies-papor-1/ 99/196 an7017 ‘Answer Key & Detaled Solutions — UPSC Civil Services Prelins Exam -2017 General Studies Paper 1 - INSIGHTS Justification: Statement 1: Arasavalli Sun Temple is a 7th-century AD Sun Temple at Arasavalli in Andhra Pradesh, Statement 3: Omkareshwar is a temple dedicated to God Shiva. https://en.wikipedia.org/wiki/Omkare: 76) Consider the following statements: 1, In the election for Lok Sabha or State Assembly, the winning candidate must get at least 50 percent of the votes polled, to be declared elected, 2. According to the provisions laid down in the Constitution of India, in Lok Sabha, the Speaker's post. goes to the majority party and the Deputy Speaker's to the Opposition. Which of the statements given above is/are correct? (a) 1 only (b) 2 only (c) Both 1 and 2 (d) Neither 1 nor 2 tp iwmwinsightsonindia.com/2017/06/1S/answer-koy-upsc-civi-services-pralims-exam-2017-general-stucies-papor-1/ so0rta6 an7017 ‘Answer Key & Detaled Solutions — UPSC Civil Services Prelins Exam -2017 General Studies Paper 1 - INSIGHTS Solution: D Justification: Statement 1: We follow the First past the post system, where majority of votes is sufficient to get elected, even if it is less than 50% of the total votes polled. Statement 2: Speaker and Deputy speaker are elected by the house. This is a convention (not constitutional provision) that usually Speaker's post goes to the majority party and the Deputy Speaker's to the Opposition. 77) Which of the following has/have occurred in India after its liberalization of economic policies in 1991? . Share of agriculture in GDP increased enormously. Share of India’s exports in world trade increased. . FDI inflows increased. . India’s foreign exchange reserves increased enormously. N aw tp iwmwinsightsonindia.com/2017/06/1S/answer-koy-upsc-civi-services-pralims-exam-2017-general-stucies-papor-1/ sovtas anra017 Answer Key & Detaled Solutions — UPSC Civil Services Prelims Exam -2017 General Studies Paper 1 - INSIGHTS Select the correct answer using the codes given below : (a) 1 and 4 only (b) 2, 3 and 4 only (c) 2 and 3 only (d) 1, 2, 3and4 Solution: B Justification: Statement 1: It has reduced post-1991. Statement 2, 3 and 4: These happened on account of liberalization of the current as well capital account. From the time when India could have defaulted BoP in 1991, we have reached a situation today where forex reserves are over $350 billion! 78. What is the application of Somatic Cell Nuclear Transfer Technology? (a) Production of biolarvicides (b) Manufacture of biodegradable plastics tp iwmwinsightsonindia.com/2017/06/1S/answer-koy-upsc-civi-services-pralims-exam-2017-general-stucies-papor-1/ so2rtas an7017 ‘Answer Key & Detaled Solutions — UPSC Civil Services Prelins Exam -2017 General Studies Paper-1 - INSIGHTS (c) Reproductive cloning of animals (d) Production of organisms free of diseases Solution: C Justification: Somatic cell cloning is a technique in which the nucleus (DNA) of a somatic cell is transferred for the generation of a new individual, genetically identical to the somatic cell donor. https://www.sciencedaily.com/terms/: 79) Consider the following statements: 1. National Payments Corporation of India (NPCTI) helps in promoting financial inclusion in the country. 2. NPCI has launched RuPay, a card payment scheme. Which of the statements given above is/are correct? (a) 1 only (b) 2 only (c) Both 1 and 2 tp iwmwinsightsonindia.com/2017/06/1S/answer-koy-upsc-civi-services-pralims-exam-2017-general- studies papor-1/ soars an7017 ‘Answer Key & Detaled Solutions — UPSC Civil Services Prelins Exam -2017 General Studies Paper-1 - INSIGHTS (d) Neither 1 nor 2 Solution: C Justification: Statement 1: NPCI is an umbrella organization for all retail payments system in India. Thus, an important objective of NPCI is to facilitate an affordable payment. mechanism to benefit the common man across the country and help financial inclusion. Statement 2: Rupay is a domestic card payment network that provides an alternative to international card schemes. It was introduced by NPCI. http://www.npci.org.in/aboutus.aspx 80. The term ‘M-STrIPES’ is sometimes seen in the news in the context of (a) Captive breeding of Wild Fauna (b) Maintenance of Tiger Reserves (c) Indigenous Satellite Navigation System tp iwmwinsightsonindia.com/2017/06/1S/answer-koy-upsc-civi-services-pralims-exam-2017-general-stucies-papor-1/ so4ta6 an7017 ‘Answer Key & Detaled Solutions — UPSC Civil Services Prelins Exam -2017 General Studies Paper-1 - INSIGHTS (d) Security of National Highways Solution: B Justification: The android-based monitoring software M-STrIPES will be used across all the Tiger Reserves of the country. http://www.newindianexpress.com/sti stripes-to-monitor-str-from-april- 1568257.html 81) What is/are the most likely advantages of implementing ‘Goods and Services Tax (GST)'? 1. It will replace multiple taxes collected by multiple authorities and will thus create a single market in India. . It will drastically reduce the ‘Current Account Deficit’ of India and will enable it to increase its foreign exchange reserves. It will enormously increase the growth and size of economy of India and will enable it to overtake China in the near future. N » Select the correct answer using the code given below: (a) 1 only tp iwmwinsightsonindia.com/2017/06/1S/answer-koy-upsc-civi-services-pralims-exam-2017-general- studies papor-1/ sosita6 an7017 Answer Key & Detaled Solutions — UPSC Civil Services Prelims Exam -2017 General Studies Paper 1 - INSIGHTS (b) 2 and 3 only (c) 1 and 3 only (d) 1, 2and3 Solution: A Justification: Statement 1: It is a standard aim of GST. Statement 2: It may reduce the fiscal deficit in the long-run and may place CAD favourably due to better foreign investment climate owing to an improved tax structure. But, the word ‘drastically’ is extreme and so the statement is incorrect. Statement 3: This is also an extreme statement and wrong. 82. ‘Broad-based Trade and Investment Agreement (BTIAY’ is sometimes seen in the news in the context of negotiations held between India and (a) European Union (b) Gulf Cooperation Council (c) Organization for Economic Cooperation and Development tp iwmwinsightsonindia.com/2017/06/1S/answer-koy-upsc-civi-services-pralims-exam-2017-general- studies papor-1/ sosita6 an7017 ‘Answer Key & Detaled Solutions — UPSC Civil Services Prelins Exam -2017 General Studies Paper 1 - INSIGHTS (d) Shanghai Cooperation Organization Solution: A Straightforward question from current affairs: it is in news since last 4-5 years. https://en.wikipedia.org/wiki/India%E 83) Consider the following statements: 1, India has ratified the Trade Facilitation Agreement (TFA) of wrTo. 2. TFA is a part of WTO's Bali Ministerial Package of 2013. 3. TFA came into force in January 2016. Which of the statements given above is/are correct? (a) 1 and 2 only (b) 1 and 3 only (c) 2 and 3 only (d) 1, 2and3 Solution: A tp iwmwinsightsonindia.com/2017/06/1S/answer-koy-upsc-civi-services-pralims-exam-2017-general-stucies-papor-1/ so7itas an7017 ‘Answer Key & Detaled Solutions — UPSC Civil Services Prelins Exam -2017 General Studies Paper 1 - INSIGHTS Why: Expected question on TFA (WTO Agreement) due to frequently being in news. Justification: Statement 1: http://economictimes.indiatimes.com/ trade/india-ratifies-trade-facilitation- agreement-of- wto/articleshow/52110279.cms. Statement 2: https://www.wto.org/english/tratop_ Statement 3: It came in force in February 2017. https://www.wto.org/english/news_ey 84. What is the importance of developing Chabahar Port by India? (a) India’s trade with African countries will enormously increase. (b) India’s relations with oil- producing Arab countries will be strengthened. (c) India will not depend on Pakistan for access to Afghanistan and Central tp iwmwinsightsonindia.com/2017/06/1S/answer-koy-upsc-civi-services-pralims-exam-2017-general- studies papor-1/ soars an7017 tpn ‘Answer Key & Detaled Solutions — UPSC Civil Services Prelins Exam -2017 General Studies Paper 1 - INSIGHTS Asia. (d) Pakistan will facilitate and protect the installation of a gas pipeline between Iraq and India. Solution: C Why: Standard question on current events of International importance Justification: The port will make way for India to bypass Pakistan in transporting goods to Afghanistan using a sea-land route. At present, Pakistan does not allow India to transport through its territory to Afghanistan. http://economictimes.indiatimes.com/ and-nation/five-things-about- chabahar-port-and-how-india-gains- from-it/articleshow/52400399.cms 85) In India, it is legally mandatory for which of the following to report on cyber security incidents? 1. Service providers 2. Data centres 3, Body corporate insightsonindia. com2017/06/19/answer-koy-upsc-civi-services-pralims-exam-2017-general-studies-paper-1/ 0916 an7017 ‘Answer Ke & Detaled Solutions —UPSC Chl Sersics Prelins Exam 2017 Genera Suis Paper- 1 - NSIGHTS Select the correct answer using the code given below: (a) Lonly (b) 1 and 2 only (c) 3 only (d) 1,2 and 3 Solution: D Justification: The CERT Rules also impose an obligation on service providers, intermediaries, data centres and body corporates to report cyber incidents within a reasonable time so that CERT-In may have scope for timely action. https://cis-india.org/internet- governance/blog/incident-response- requirements-in-indian-law 86. Right to vote and to be elected in India isa (a) Fundamental Right (b) Natural Right (c) Constitutional Right tp iwmwinsightsonindia.com/2017/06/1S/answer-koy-upsc-civi-services-pralims-exam-2017-general- studies papor-1/ snortas an7017 ‘Answer Key & Detaled Solutions — UPSC Civil Services Prelins Exam -2017 General Studies Paper 1 - INSIGHTS (d) Legal Right Solution: D Justification: There is dispute whether the answer should be C or D. Case for Option C: Logically the answer seems to be C because Article 326 of constitution grants adult suffrage which implies the right to vote. Moreover, a democratic constitution loses its essence if it does not provide for a right to vote as a constitutional right. It is true that Representation of People’s Act also provides for a Right to Vote, but they may be considered qualifications (conditions) on the Right to Vote already granted by the constitution in A326. ‘An analogy can be Child Labour Act or Bonded labour prohibition Act where these fundamental rights are separately enforced by law. So, the case for Option C looks strong. Option D: But, the Supreme Court is the final interpreter of the constitution and its view will prevail notwithstanding all arguments. tp iwmwinsightsonindia.com/2017/06/1S/answer-koy-upsc-civi-services-pralims-exam-2017-general-stucies-papor-1/ sits ani7a0%7 ‘Answer Key & Detailed Solutions - UPSC Chil Services Prelims Exam -2017 General Studies Paper- 1 - INSIGHTS The Supreme Court observed in PUCL. vs. Union of India (2015), “No doubt, the right to vote is a statutory right but it is equally vital to recollect that this statutory right is the essence of democracy. Without this, democracy will fail to thrive, Therefore, even if the right to vote is statutory, the significance attached with the right is massive,” Therefore, the answer to this question is D, not C. http://www. pucl.org/Topics/Law/2012 87. What is the purpose of ‘evolved Laser Interferometer Space Antenna (eLISA)’ project? (a) To detect neutrinos (b) To detect gravitational waves (c) To detect the effectiveness of missile defence system (d) To study the effect of solar flares on our communication systems Solution: B Justification: eLISA’s primary objective is the detection and examination of gravitational waves tp iwmwinsightsonindia.com/2017/06/1S/answer-koy-upsc-civi-services-pralims-exam-2017-general-stucies-papor-1/ snares ani7a0%7 ‘Answer Key & Detailed Solutions - UPSC Chil Services Prelims Exam -2017 General Studies Paper 1 - INSIGHTS emitted by the super-massive black holes that reside in the centres of many galaxies. In addition, eLISA will measure the signals of thousands of compact binary star systems in the Milky Way. https://en.wikipedia.org/wiki/Laser_Ir 88) What is the purpose of Vidyanjali Yojana’? 1. To enable the famous foreign educational institutions to open their campuses in India. 2. To increase the quality of education provided in government schools by taking help from the private sector and the community. 3. To encourage voluntary monetary contributions from private individuals and organizations so as to improve the infrastructure facilities for primary and secondary schools. Select the correct answer using the code given below : (a) 2 only tp iwmwinsightsonindia.com/2017/06/1S/answer-koy-upsc-civi-services-pralims-exam-2017-general- studies papor-1/ snanas an7017 Answer Key & Detaled Solutions — UPSC Civil Services Prelims Exam -2017 General Studies Paper 1 - INSIGHTS (b) 3 only () 1 and 2 only Solution: A Why: Lifted from MHRD‘s website home page, as is the case for most. government schemes that have been lifted from the official websites in this paper. Justification: This programme has been envisaged to bring together people willing to volunteer their services at schools which really need them. The aim of the programme is to strengthen implementation of co- scholastic activities in government. schools through services of volunteers. http://vidyanjali.mygov.in/index.php/f 89. What is the aim of the programme *Unnat Bharat Abhiyan’? (a) Achieving 100% literacy by promoting collaboration between voluntary organizations and tp iwmwinsightsonindia.com/2017/06/1S/answer-koy-upsc-civi-services-pralims-exam-2017-general- studies papor-1/ anes anra017 ‘Answer Key & Detaled Solitons - UPSC Gil Seices Prelims Exam 2017 General Stusies Pope 1 = NSIGHTS government's education system and local communities. (b) Connecting institutions of higher education with local communities to address development challenges through appropriate technologies. (c) Strengthening India’s scientific research institutions in order to make India a scientific and technological power. (d) Developing human capital by allocating special funds for health care and education of rural and urban poor, and organizing skill development programmes and vocational training for them. Solution: B Why: Lifted from MHRD’s website: Government scheme, Justification: MHRD has launched a programme called Unnat Bharat Abhiyan with an aim to connect institutions of higher education, including Indian Institutes of Technology (IITs), National Institutes of Technology (NITs) and Indian Institutes of Science Education & Research (IISERs) etc. with local communities to address the development challenges through appropriate technologies. tp iwmwinsightsonindia.com/2017/06/1S/answer-koy-upsc-civi-services-pralims-exam-2017-general-stucies-papor-1/ 15/196 an7017 Answer Key & Detaled Solutions — UPSC Civil Services Prelims Exam -2017 General Studies Paper 1 - INSIGHTS http: /unnat.jitd.ac.in/index.php/en/ 90) Consider the following statements : 1. The Election Commission of India is a five-member body. . Union Ministry of Home Affairs decides the election schedule for the conduct of both general elections and bye-elections. . Election Commission resolves the disputes relating to splits/mergers of recognised political parties. N w Which of the statements given above is/are correct ? (a) 1 and 2 only (b) 2 only (c) 2 and 3 only (d) 3 only Solution: D Why: The recent controversy surrounding Samajwaadi Party split/merger in UP elections where ECI recognized the party led by tp iwmwinsightsonindia.com/2017/06/1S/answer-koy-upsc-civi-services-pralims-exam-2017-general-stucies-papor-1/ wnena6 an7017 Answer Key & DetoledSolstons -UPSC Chil Sences Prelims Exam 2017 General Stes Paper-1 = INSIGHTS Akhilesh yadav as the one representing SP. Justification: Statement 1: Presently it is a three member body, one CEC and two other ECs. Statement 2: ECI decides this schedule, Statement 3: Splits, mergers and alliances have frequently disrupted the compositions of political parties. This has led to a number of disputes over which section of a divided party gets to keep the party symbol, and how to classify the resulting parties in terms of national and state parties. The Election Commission has to resolve these disputes, although its decisions can be challenged in the courts. http://eci.nic.in/eci_main1/the_functic 91. In India, if a species of tortoise is declared protected under Schedule I of the Wildlife (Protection) Act, 1972, what does it imply ? tp iwmwinsightsonindia.com/2017/06/1S/answer-koy-upsc-civi-services-pralims-exam-2017-general-stucies-papor-1/ wires an7017 ‘Answer Key & Detaled Solutions — UPSC Civil Services Prelins Exam -2017 General Studies Paper 1 - INSIGHTS (a) It enjoys the same level of protection as the tiger. (b) It no longer exists in the wild, a few individuals are under captive protection; and now it is impossible to prevent its extinction. (c) It is endemic to a particular region of India. (d) Both (b) and (c) stated above are correct in this context. Solution: A Justification: Schedule I animals under the Wildlife Act enjoy the highest protection in India. Since tiger is also covered in Schedule I, option A is correct. www.envfor.nic.in/legis/wildlife/wildlif 92. In India, Judicial Review implies (a) the power of the Judiciary to pronounce upon the constitutionality of laws and executive orders. tp iwmwinsightsonindia.com/2017/06/1S/answer-koy-upsc-civi-services-pralims-exam-2017-general-stucies-papor-1/ rans anra017 Answer Key & Detaled Solutions — UPSC Civil Services Prelims Exam -2017 General Studies Paper 1 - INSIGHTS (b) the power of the Judiciary to question the wisdom of the laws enacted by the Legislatures. (c) the power of the Judiciary to review all the legislative enactments before they are assented to by the President. (d) the power of the Judiciary to review its own judgements given earlier in similar or different cases. Solution: A Justification: Judicial Review refers to the power of the judiciary to interpret the constitution and to declare any such law or order of the legislature and executive void, if it finds them in conflict the Constitution of India. 93) With reference to Indian freedom struggle, consider the following events : 1. Mutiny in Royal Indian Navy 2. Quit India Movement launched 3, Second Round Table Conference tp iwmwinsightsonindia.com/2017/06/1S/answer-koy-upsc-civi-services-pralims-exam-2017-general-stucies-papor-1/ ni9ita6 anra017 Answer Key & Detaled Solutions -UPSC Cl Senices Prelim Exam 2017 General Sie Paper-1 - INSIGHTS What is the correct chronological sequence of the above events ? (a) 1-23 (b) 2-1-3 (324 (d) 3-12 Solution: C Justification: Second Round Conference happened in 1931, Quit India Movement in 1942 and Mutiny in Royal Indian Navy in 1946. 94) Consider the following statements : 1. Tax revenue as a percent of GDP of India has steadily increased in the last decade. 2. Fiscal deficit as a percent of GDP of India has steadily increased in the last decade. Which of the statements given above is/are correct ? (a) 1 only (b) 2 only tp iwmwinsightsonindia.com/2017/06/1S/answer-koy-upsc-civi-services-pralims-exam-2017-general-stucies-papor-1/ 206 anra017 tp iwmwinsightsonindia.com/2017/06/1S/answer-koy-upsc-civi-services-pralims-exam-2017-general-stucies-papor-1/ ‘Answer Key & Detaled Solutions — UPSC Civil Services Prelins Exam -2017 General Studies Paper-1 - INSIGHTS (c) Both 1 and 2 (d) Neither 1 nor 2 Solution: D Why: Basic macroeconomic data from Economic Survey Justification: Statement 1: It asks about India’s Tax buoyancy, which the responsiveness of tax revenue mobilisation to GDP growth. Tt is calculated as a ratio of percentage growth in tax revenues to growth in nominal GDP for a given year. Tax is said to be buoyant (more than 1) if the gross tax revenues increase more than proportionately in response to a rise in GDP figures. If it is less than 1, it means tax revenues have not increased proportionately with GDP growth. The figure below shows variation in tax buoyancy from 2004-05 to 2015- 16, which have been up and below 1 like a wave. So, statement 1 is wrong. The graph above shows tax revenues as a percentage of GDP from 2002 to 2014. rans an7017 ‘Answer Key & Detaled Solutions — UPSC Civil Services Prelins Exam -2017 General Studies Paper-1 - INSIGHTS https://thewire.in/32648/new-data- indicates-that-direct-tax-share-in- economy-is-at-decade-low/ Statement 2: Fiscal deficit as a percentage of GDP had increased rapidly in recession years (post 2008) due the fiscal stimulus given by the government and then declined due to the fiscal consolidation measures adopted. So, 2 is also wrong. http://www.thehindubusinessline.com buoyancy-improves-thanks-to- indirect-levy/article9466851.ece tp iwmwinsightsonindia. com/2017/06/1S/answer-koy-upsc-civ-services-prolims-exam-2017-goreral-slusios-paper-t/ 26 an7017 ‘Answer Key & Detaled Solutions - UPSC Givi Series Palins Exam -2017 General Stsies Paper~1 - INSIGHTS 95. Recently there was a proposal to translocate some of the lions from their natural habitat in Gujarat to which one of the following sites ? (a) Corbett National Park (b) Kuno Palpur Wildlife Sanctuary, (c) Mudumalai Wildlife Sanctuary (d) Sariska National Park Solution: B Why: In news and mired by controversies due to Gujarat government's reluctance to send tigers to MP. The Supreme Court in 2013 ruled in favour of translocation of the Asiatic lion to Kuno in the interest of the genetic stability of the species. http://economictimes.indiatimes.com/ and-nation/gujarat-swallows-its- pride-may-agree-to-translocate-gir- lions-to-kuno- sanctuary/articleshow/52803382.cms tp iwmwinsightsonindia.com/2017/06/1S/answer-koy-upsc-civi-services-pralims-exam-2017-general- studies papor-1/ raanas anra017 ‘Answer Key & Detaled Solutions — UPSC Civil Services Prelins Exam -2017 General Studies Paper-1 - INSIGHTS 96) Which of the following are not necessarily the consequences of the proclamation of the President's rule in a State? . Dissolution of the State Legislative Assembly . Removal of the Council of Ministers in the State 3. Dissolution of the local bodies N Select the correct answer using the code given below: (a) 1 and 2 only (b) 1 and 3 only (©) 2 and 3 only (d) 1, 2 and 3 Solution: B Justification: Statement 2: When the President's Rule is imposed in a state, the President dismisses the state council of ministers headed by the chief minister. Statement 1: The President can either suspend or dissolve the Legislative assembly, so 1 is not necessarily the consequence of proclamation. tp iwmwinsightsonindia.com/2017/06/1S/answer-koy-upsc-civi-services-pralims-exam-2017-general-stucies-papor-1/ s2ana6 anra017 ‘Answer Key & Detaled Solutions — UPSC Civil Services Prelins Exam -2017 General Studies Paper-1 - INSIGHTS 97) Which of the following are envisaged by the Right against Exploitation in the Constitution of India? . Prohibition of traffic in human beings and forced labour . Abolition of untouchability Protection of the interests of minorities . Prohibition of employment of children in factories and mines wn s Select the correct answer using the code given below: (a) 1, 2 and 4 only (b) 2, 3 and 4 only (©) 1 and 4 only (d) 1, 2, 3.and 4 Solution: C Justification: Abolition of untouchability comes under Right to Equality. Protection of the interests of minorities comes under Cultural and Educational Rights. tp iwmwinsightsonindia.com/2017/06/1S/answer-koy-upsc-civi-services-pralims-exam-2017-general-stucies-papor-1/ s2sita6 an7017 ‘Answer Key & Detaled Solutions — UPSC Civil Services Prelins Exam -2017 General Studies Paper 1 - INSIGHTS 98. Which of the following is geographically closest to Great Nicobar ? (a) Sumatra (b) Borneo (c) Java (d) Sri Lanka Solution: A Justification: Distance from Great Nicobar to Sri Lanka is 1,437 km. Distance from Great Nicobar to Sumatra is 1,192 km. tpn 99. Out of the following statements, choose the one that brings out the principle underlying the Cabinet form of Government: (a) An arrangement for minimizing the criticism against the Government. whose responsibilities are complex insightsonindia. com/2017/06/19/answer-koy-upsc-iv-services-prolims-exam-2017-goreral-slusios-paper-t/ 12696 anra017 Answer Key & Detaled Solutions -UPSC Cl Senices Prelim Exam 2017 General Sie Paper-1 - INSIGHTS and hard to carry out to the satisfaction of all. (b) A mechanism for speeding up the activities of the Government whose responsibilities are increasing day by day. (c) A mechanism of parliamentary democracy for ensuring collective responsibility of the Government to the people. (d) A device for strengthening the hands of the head of the Government whose hold over the people is in a state of decline. Solution: C Straightforward question 00. Which one of the following is not a feature of Indian federalism? (a) There is an independent judiciary in India. (b) Powers have been clearly divided between the Centre and the States, (c) The federating units have been given unequal representation in the Rajya Sabha. tp iwmwinsightsonindia.com/2017/06/1S/answer-koy-upsc-civi-services-pralims-exam-2017-general-stucies-papor-1/ arias an7017 Answer Key & DetoledSolstons -UPSC Chil Sences Prelims Exam 2017 General Stes Paper-1 = INSIGHTS (d) It is the result of an agreement among the federating units. Solution: D Justification: The phrase ‘Union of States’ has been preferred to ‘Federation of States’ for two reasons: one, the Indian Federation is not the result of an agreement among the states like the American Federation; and two, the states have no right to secede from the federation. The federation is an Union because it is indestructible. The country is an integral whole and divided into different states only for the convenience of administration. Please Share this Post © Telegram (aaa) (Dwhatien) c+ < More [OFFICIAL DOWNLOAD DETAILED ANSWER Question UPS KEYS-2013] Paper: UPSC Preparation upsc Civil Services Strategy: tp iwmwinsightsonindia.com/2017/06/1S/answer-koy-upsc-civi-services-pralims-exam-2017-general-stucies-papor-1/ 2an6 an7017 tpn ‘Answer Key & Detaled Solutions — UPSC Civil Services Prelins Exam -2017 General Studies Paper 1 - INSIGHTS Releases 2013 IAS Chandra Preliminary Mohan Garg, Exam 2016 Rank - 25, ‘Answer Keys August 7, UPSC Civil une 26, 2014 2016 Services Exam In "2016 = 2015 STRATEGY" May 22, 2016 In "TOPPERS' Tags: answer key upsc ias prelims paper insights answer key 2017 prelims insights ias answer key prelims os paper 1 paper I ias upsc preims answer key Ups ias prelims insights ias answer key PREVIOUS NEXT STORY STORY ONLINE Insights APPLICATION: < Daily Insights Current ofine > Affairs, 19 Guidance June 2017 Program — 2018 Comments Community @ Login ~ Recommend +4 [Share Sort by Best » Join the discussion. Loc In with oR SIGN uP with pisaus (2) nsightsonindia. com/2017/06/19/answer-koy-upsc-iv-services-prolims-exam-2017-goreral-slusios-paper-t/ s29n6 an7017 ‘Answer Key & Detaled Solutions — UPSC Civil Services Prelins Exam -2017 General Studies Paper 1 - INSIGHTS Name e Bobde dada + 2 months ago =_ 74.0 + Reply + Share> Ashish Yadav 4 Bobde dada * 2 months ago tp iwmwinsightsonindia.com/2017/06/1S/answer-koy-upsc-civ-services-prolims-exam-2017-goreral-slusios-paper-t/ sa0rta6 an7017 ‘Answer Key & Detaled Solutions — UPSC Civil Services Prelins Exam -2017 General Studies Paper-1 - INSIGHTS He might be having dew with kitkat 23.0 v + Reply + Share> @ Bashers Genus - —™ Berserk) > Ashish Yadav + 2 months ago hahahaha 2A v + Reply + Share» @ Anurag Basu + Bote dada * 2 months ago oh teri, books ke alava pehle namkeen, biscuit, chocolates etc khatm karlo bhai log 18.A v © Reply » Share» @ Bebe: aaaae Anurag Basu +2 months ago Kuda, karkat, walls kuch bhi. Mat chodo 9A v= Reply + Share > @ sapesin sack Sparrow > = Anurag Basu + 2 months ago or lipstick me lead. hahaha,...dimag kharab kr denge ye ek din....thank god hota h...lipstick me lead 3A v + Reply + Share > @ reves Azuis -=—s Captain JACK Sparrow +2 ‘months, ago final canelusion tp iwmwinsightsonindia. com/2017/06/1S/answer-koy-upsc-civ-services-prolims-exam-2017-goreral-slusios-paper-t/ savas an7017 ‘Answer Key & Detaled Solutions — UPSC Civil Services Prelins Exam -2017 Genera Studies Paper 1 - INSIGHTS kya hai ..lead hai ya nahi QA 1A ve Reply « Share > Captain JACK ‘Sparrow a Trevias Anis +2 months ago hai...bilkul h....hadd h ye kisi bhi ladhki ko nhi pta Reply + Share > Punam a Captain JACK Sparrow *2 months ago There is lead in lipsticks and mercury too, There was an article in the Hindu saying that LOréal had more of these in their products. BA ye Roply + Share > Captain yack sparrow 2 Punam 12 months 290 haan mujhe pia h....kisi tact earlee ba hp iwmwinsightsonindia. com/2017/06/1S/answer-koy-upsc-civ-services-prolims-exam-2017-gareral-slusios-paper-t/ sa2ta6 an7017 ‘Answer Key & Detaled Solutions — UPSC Civil Services Prelins Exam -2017 General Studies Paper-1 - INSIGHTS hique hye bhi....dekha tha...ek bar. *) tAalve Reply » Share» THE REVENANT > Captain JACK Sparrow “2 ‘months ago boys should know it better tAaive Reply + Share > Captain wack Sparrow A THE REVENANT “2 months 90 pta nhi bhaiya....we are in single forever category. 2a ve Reply « Share > yom Yadav a Captain JACK Sparrow “2 months ago Biwi lipstick mange to ye question padha dio usko :p taive Reply « Share > tp iwmwinsightsonindia. com/2017/06/1S/answer-koy-upsc-civ-servicesprolims-exam-2017-goreral-slusios-paper-t/ saata6 ani7a0%7 ‘Answer Key & Detailed Solutions - UPSC Chil Services Prelims Exam 2017 General Studies Paper- 1 - INSIGHTS sw yACK = Sparrow Ayo Yadav +2 months ago jaise UPSC paper set krra h pre ka....2-3 saal se...is hissab se na biwi milegi na job... +) tAive Reply » Share > @ ws Yadav =e ATE REVENANT +2 montis ago Kiss kat kario lipstick wali ko ave Reply « Share > @ REVENANT <2 Pvyom Yadav +2 ‘months ago u from UP or bihar ? Reply + Share > @ we Yadav ee THE REVENANT °2 ‘months ago Haryana 2a ve Reply - Share» THE REVENANT yom Yadav +2 ‘months bpiiwmw insightsonindia.com/20'7106/1Sianswer-key-upsc-civi-services-prelims-exam-2017-generab-slucies-paper-t/ s3art96 an7017 ‘Answer Key & Detaled Solutions — UPSC Civil Services Prelins Exam -2017 General Studies Paper 1 - INSIGHTS ago Buddha ne bhi yehi sochke allow nhn kiya hoga apne vinara main Reply + Share > THE REVENANT A vyom Yadav +2 ‘months ago UPSC ka bhala ho aankhein khol di Reply + Share» Try one more time! > Trevias Azuis +2 ‘months ago hota hai :p Ave Reply - Share > Trevias Aauis aiy ‘more time! “2 ‘months ago koi thora sa zeher de do seriously @ BBic nw tp iwmwinsightsonindia.com/2017/06/1S/answer-koy-upsc-iv-services-prolims-exam-2017-goreral-slusios-paper-t/ sasta6 an7017 Answer Key & Detaled Solutions — UPSC Civil Services Prelims Exam -2017 General Studies Paper 1 - INSIGHTS tp iwmwinsightsonindia.com/2017/06/1S/answer-koy-upsc-iv-services-prolims-exam-2017-goreral-slusios-paper-t/ 36096

Anda mungkin juga menyukai